You are on page 1of 69
‘Note: This material is copyrighted. All rights reserved. Edward Goljan, M.D. 2003, PATHOLOGY QUESTIONS ‘Note: This material is copyrighted. All rights reserved. (Eaward F, Goljan, M.D.) 2002. ‘HIGH YIELD STUDY QUESTIONS IN PATHOLOGY Optional: General principles im laboratory medicine questions: this material is covered in the Pathology High Yield section 1, Assuming theuse of 2 standard deviations to establish the reference interval of a test, in atest with ‘reference interval of 10-30 mg/L, 1 standard deviation would equal. a 3S B50 & 95 D. 100 E209 Answer: B-mean ofthe tests 20 mg/dl, 2 SD ~ 10 mg/l, therefore 1 SD = mp/al. 2. Ifthe prostate specific antigen (PSA) test for prostate cancer is lowered from a referenes interval of 9-10 to 0-4 nl i ‘increase the number of false negatives B Gonean Sane of Wat pote C__increase the test's specificity D, increase the PV" , _inerease the PY" ‘Answer: D~ increasing sensitivity (less FNs) by lowering the reference interval automatically inereases the predictive value of anegative test result 3, Study the following schematic involving a control group and disease X. ‘Disease X (Ke Reference interval ‘Which ofthe following correctly deseribes test results in the space occupied by each of the lettered oups? ‘Group A: true negaives + false negatives ‘Group B: true nogaives * false positives ‘Group C: true positives + false positives ‘Group D: true positives «false negatives ‘group A™= all TNs, proup B= TNs +FNs, group C=FPe + TPs, group D= all TPs 4, A pregnant woman in her first trimester has an elevated serum T, and normal TSH. Physical exam ‘smormal. Which of the following best explains the thyroid funotion study results? ‘Thyroid binding globulin i increased Free'T, hormone levels are increased “Estrogen increased the synthesis of thyroid hormone Progesterone inereased the synthesis of thyroid binding globulin Patient has Grave's disease Answer: A~ estrogen increases the synthesis of thyroid binding globulin. ‘This automatically inereases the total Ty level, since TBG has T bound to it. However, the free 1 level remains unchanged. A. similar question could be an inerease in serum cortisol in a pregnant woman who has no signs of (Cushing's syndrome. Estrogen also inereases the synthesis of transcortn, the binding protein for cortisol, hence the increase in serum cortisol, but no signs of hypercortisolism sinee free eartisol levels are normal. POORP ‘Note: This material is copyrighted. All rights zeserved. (Edward F. Goljan, M.D.) 2002 Cell Injury questions ‘Hyperplasia sprint operative in which of the ftlowing growth alterations? “Appeatinee of the affected Kidney in renovascular hypertension ‘Thickened bladder wall ina patient with wettal obstruction ‘Baret!sesophagas in a patient with gastruesophageal efx Enlarge left atrium in pation with severe mitral stenosis Galactothea ina woman with prolactinorsa Asner: E- endosrne stilton of aget sues is invasably hyperplasi. Choice A is atophy, choice 1Bas hypertophy, choice Cis metaplasia, choice D is hypertophy 2. Which ofthe folowing disorders is an example of cougulation necrosis? A. Lobar psumona nan alsohoie [B, Hepatioubseess in x patient with amcbisis 1G. Peeudomerbranous cots in patient on arpiilin D. Diminished brain mass a patent with Alhelmer’s disease 'E, _Embolus tothe superior mesenteric artery leading to bowel infection Answer: E-bemorthagi infection. Choices A, B, and C ae liguefative necrosis. Chaice Dis atrophy, 3. Ia which of he following discases would you expect low arterial PO, and. low oxygen saturation Peop> (S20)? ‘A. Carbon monoxide potsoning B, Iron deficiency anemia CC. Decreased cardiac ontpat D. Respiratory acidosis E. _ Cyanide poisoning Answer: D~choive A has a low SO», choices B, C, and E have a normal PO, and Sa0>, choice 4. Which of the following disorders issn example of metaplasia? A. Inereased goblet cells in the mainstem bronchus of a smoker B. Squamous epithelium in the mainstem bronchus ofa smoker C.Prolifertive endometrial gland in a woman on unopposed estrogen. D. _ Hypetkeratosis ofthe kin in «patient with psoriasis, E._ Multinusteated giant calls ina granniome ‘Answer: B~ goblet cells in the mainstem bronchus are an example of iyperplasia. If they were in the terminal bronchiole, it would he metaplasia. Choices C and D are hyperplasia. Choice E represents granulomatous inflammation, 1, A deyeanold child with recuinent Staphylococcus aureus infectious and an absent respiratory burst ‘MOST LIKELY bas alan... ‘A. defect in spectrin i the coll membrane D. defect in microtubule polymerization ©. deficiency of IgG gamma globulins D. — deficiensy of NADPH oxidase LE, _ deficiensy of myeloperoxidase Answer: D- patient with MPO deficiency would bave a respiratory burst but would still have a ‘microbicidal problem. In addition, there ate no azurophilic granules in the cytoplasm of neutrophils. 2 _"Whien ofthe followings MOST epost for the amt-inlammatry ivy of onsets? Increased leukocyte adhesion to endothelial cells Inhibition of phospholipase Az Destruction of eosinophils Inhibition of cyclooxygenase poe (Note: This material is copyrighted. All ights reserved. (Baward F. Golan, MID.) 2002 E. _ Inhibition ofpaxygemaso inhibition of PLA, leads toa reduction in synthesis of both prosaplandins and Teukotienes. A. Cellulitis B. Suppurative inflammation Cl Fibrinous inflammation, D. _Psevxfomembranons inflammation , Granulomatous inflammation 3. A.S2.yearald man with eronis renal failure has chest pain and a srtchy 3 component sound ica over the anterior chest Answer: Chis is pericardial fet aA fbi Sear od il, ho hs nocrocetved any immuniarons, has a grayish. wite exw the oropharyn sod prominest cevielIymphadenopaty Answer: D=nove, C.ifite pseudomerranns colts fas a ot of simatic wt ihtheria ‘S.A febrile 3-year-old child has a diffuse, raised, hot red area of inflammation extending around the ‘ight perioral Gane leading fo swelling and lone ofthe eye Answer: A~ cellulitisis usually due to group «A strep, since hyaluronidase (spreading factor) is produced sn allows th infcetion fo spread rg the mibetancous fase. Sty ococeus mes proces coagulase, which changes fibrinogen into fibrin and locelizes the infection leading; tw an abstess ‘eit anager the curfac through «sis. 6. A newbom child has failure of separation of the umbiliéal cord. Histologic sections of the surgically removed cord reveal sn absence of neutrophil margination and emigration info the interstitial tissue. The clinical and histologic findings in this case are MOST CLOSELY associated witha defect in. ‘A activation of the complement system B, _orotubule polmeriation © therespiratory burst mechanism 1 sadhesion mols in nexenils the icustin of mcg 1D- without adhesion molecules, neuropil camot adhere to the endotlel ells in the ‘eles, henee they cannot emigrate into tse: For the umbilical cord to fll of it mst ave "ettropalnfiktion of the tess leading to nero Answé ems 7-9 A. Neutrophils B. Macropiages ©. Eosinophils 1D. Mastctls E. Plasmacells 7. ‘They represent the epithelioid cells and multinucleated giant ells ina granuloma Answer: B- they are activated by 7-interferon released from helper T cells 8. CSaand C5a activate these cells “Answer: D~ eauses them to release preformed chemicals like histamine and serotonin 9. The granules of these cells contain reffactile material that form Charcot-Leyden erystals in the sputum of asthmatios “Answer: C- recall that eosinophils have crystalline material in their red granules ‘Fhulds and hemodynamics questo T. Which of the following characterizes early endotoxie (septic) shock rather then hypovolemic or cardiogenic shock? A. Warm skin "Note: This meters] is copyrighted, All rights reserved. (Edward F. Goljan, M.D.) 2002 B, Decreaced cardiac output . Increased total peripheral resistance D. Decreased venous return to the heart Answer: A~ due to the decrease in TPR from vasodilatation (histamine, bradykinin, nitric oxide, release of anaphylatoxins) 2. Which of the following edema conditions represents a transudate secondary to an decrease in ‘nootie pressure? ‘A. Patient with pneumonia who has a pleural effusion B. Patient with cithosis who las dependent pitting edema (©. Palient with edema ofthe arm post-modified radical mastectouny 1D. Patient with « pulmonary infarction who has a lft pleural effusion .E. _ Patient with congestive heart failure with bilateral pleural effusions Answer: Bin cihosis, there are 2 afterations in Starlings forces an increase in hydrostatic pressure ‘fom portal vein Hypertension and a deorease in synthesis of albumin, which decreases oneotie pressure, Choices A, and D are examples of exudates. Choice Cis lymphedema. Choice E is an ‘increase in hydrostatic pressire causing a ransudate 3. In treating potiont with right-sided heart filure who has dependent pitting edema, which of the {following would be the MOST APPROPRIATE management of the patient's sodium and water intake? ‘Sodium intake Water intake A. Nochaoge Deerease B, Tacrease Increase C. Deorense No change D. Decrease Desrease E_ Nochange No change ‘Answer: D patients with heart feilure and pitting edema have an excess of salt and water, hence these ‘st berestricted. 4. A S8yearold man with a small cell carcinoma of the lung presents with mental status ‘shoormalitis. ACT scan of the brain reveals cerebral edema but no space oocupying lesions. ‘Serum electrolytes exhibit a serum sodium of 110 mEq/L (136-145 mg/L). There is no evidence ‘of pitting edema or volume depletion. Which of the following is the BEST non-pharmacologie ‘reaument ofthis patient? ‘Sodium intake Water intake A. Deeresse Decrease Be Increase Increase Decrease No change DB. Nochenge Decrease E Nochange No change Answer: D- the patient has inappropriate ADH syndrome. Since ADH reabsorbs solute free water from the Kidneys, the excess water ditributes in both the ECF and ICF compartments. Dilutional hyponatremia favors an osmotic gradiemt moving water into the ICF compartment. The Rx of eboice is to restrict wate, since the TTBNa is normal. 5. 62-year-old man has a ruptured abdominal aortic aneurysm with retroperitoneal hemorrhage. ‘Which of he following findings BEST represents ths patient clinical disorder? LVEDP TPR Cardiac output Normal Normal Normal Tnereased —Inereased ~—_Dereased Decreased Increased Decreased Decreased Decreased Decreased ventricular end-listolic pressure), TPR = total peripheral arteriolar resistance) 4 fote: This material is copyrighted. All vights reserved, (award F. Goljan, M.D.) 2002 Answer: C- the patient isin hypovolemie shock, hence the cardiac output it decreased, TPR is increased ‘due to catecholamines, vasopressin, and ATT T, and the LVEDP is decreased, since plasma volume is decreased, LYEDY TPR —Cardine output Interpretation A. Normal = Normat_——_-Normal Nocmal B. Tnoteased _teease_ Decreased Tefe-sided ear fitore D, Decreased Decreased Decreased Septic shock 6. Which ofthe following i a coagulation factor that is utilized it both an occlusive venous thrombus and in.an arterial thrombus? A. Factor XI B, Factor VIE Factor ne D. FactorX E, _ Fibrinogen “Answer: E- fibrinogen is present in both venous thrombi (Abin clot trapping RBCs, plstelts, WBCS) and in atc thromb (sgpegntd patlets held oyster by Bitrn), 7 429 yr, old man sustain bilateral femoral bone fuctures and mulliple pelvic fractures from 2 rmouniain bike accident. Forty eight hour later, he develops a sudden onset of dyance, petechiel lesions, and mental stats alterations. Laboratory stes reveal hypoxemia and thrombocytopenia ‘The mechanism for this patient’ clinical disorder ismost closely associated with. ‘A. dissented intravascular coagulation B. pulmonary enoliam Cfatembolization D. sicembolization pneumonia “Answer: C- the patent has ft embolization, Note the time delay before symptoms occur. ‘Nutrition questions: optional. Material in High Yield “A 25ayearald woman has not had her period forthe last 8 months, She is $* 2" and weighs 90 ‘pounds. A urine pregnancy testis negative. Se states that she fas been trying 10 lose weight for her “upeoming wedding. You order a batery of tests and give the patient an intramuscular injection of ‘progesterone. Ten days later the patient retus to your office and reports that she had no ilhdcawal bleeding, Laboratory tests reveal the following: serum prolaet is normal, serum FSH and LH are low, serum TSH is normal, seram estradiol is low, and serum cortisol an growth Ihormone are inereased, Based on these findings, you strongly suspect the patient has. ‘A. primary ovarian disease B. hypopituitarism. ©. secondary hypothyroidism. D. eight loss syodrome BE. _ Cushing's syndcome ‘Answer: D= anorexia nervosa. Amenorrhea leaves the patient estrogen deficient and prone to ‘osteoporosis, Ste should be placed on birth contol pills. Note how the stress hormones (cortisol, GH are increased. 2. Follicular hyperkeratosis, night blindness, and a hemorshagic diathesis are expected in a patient... - with eystic fibrosis with scurvy ‘vith hypothyroidism ho is pure vegan ‘whois bulimic FOOE> [Notes This material is copyrighted, Al rights reserved, (Edward F. Goljan, M.D.) 2002 ‘Answer: A~vitamin A deficiency accounts for the first 2 symptoms, while vitamin K deficiency accounts for the hemorrhagic diahesis. Patients with CF have malabsorption and lack all the Tat soluble vitamins. 3. Which of the signs or symptoms characterize a fat soluble rather than a water soluble vitamin * Seficiency? ‘A. Pecflliculr hemorrhage B. Bone pain and tetany © Peripheral. D. Ophiimopleia, confusion, and ataxia E_Hyperpigmenitionin sun-exposed areas Answer: B- vitamin D defisiency causes be pun and tetany, since the bone is not mineralized properly ‘andi soft (osteomalacia) eaving i prone to fractures. Hypoosicemia explains the tian. Choice A {is seury, choice Cis pyridoxine deficiency, choice Dis thiamine deficiency, choice Es pellgra fom niscin deficiency. Which of tho sign or symptoms are mare prominently found in marasms than kwachioxkor? A. Hepatomegaly B. iting edoms ©. Redused total Iyaphoeyte count 1D, Hlsky-pain dermatitis B._ Broomstick extremities ‘Answer: Ball the other findings eharacterize kwashiorkor, whe there is anormal total calorie insko ‘but dfiieney of protein 5. Pellagra will MOST LIKELY develop in a patient. ‘A. whois taking isoniazid 1B. whose diet primarily consists of com who isa pare vegan D. whos taking nicotinic ac to lower lipids E, _withmaldigeston evondary to chronic pneeatiis Answer: B~ com contains niacin which isin a bound form that extnot be absorbed, Choice A is Dytdoxine deficiency, choice Cis By defcienoy, choice D isa by-product of niacin, choice rnaldgestion will nuse ft soluble vitamin defiicnces 4, D, EK. 6. Which ofthe following is more often associated with anorexia nervose than bulls necvosa? A. Bypolalemia B. Metabolic alkalosis Normal body image D. Nonna serum gonadotropins E. Osteoporosis “Answer: 1~ due toa lack of estrogen. All the ocr findings are moce commonly seen in bulimia. 7. Which of the following laboratory test abnormalities would you MOST expect ia patent with ‘morbid obesity? ‘A. Increased serum TSH BL Inereased 24-hr urine for fre cortisol C._Inereased fasting glucose 1D. Increased 24-hr urine for 17-kotosteroids E. _Inereased secum DHEA-sulfate Answer: C-increased adipose down-tegulates insulin receptor synthesis predisposing the patient to type TI DM or glucose intolerance. A urine for free cortisol clearly separates obesity from Cushing's ‘syndrome, the latter having an increase in free levels inthe urine. 8. Which ofthe following vitemin deficiencies would you expect in child maintained on unfortified ‘goat's milk? AL Ascorbic acid ‘Note: This material is copyrighted. All rights reserved. (Baward F. Goljan, MD) 2002 ‘Thiamine Niacin Riboflavin Folate nove ont miki defi ete nd yeni ‘Which oF the following vitamins would be Jeficint in ¢mewora child with anemia whose mother isapure vegan? “Ascorbic acid ‘Thianine Niacin Pdi ‘Answer: J pate vegans donot eat meat or diry products, hence they are susceptible wo Bs deficieny. ‘Vegans who are piregnant should be placed on Biz supplements in addition to the usual prental ‘tasns containing folate and ire. 10. A.65.yearold-woman complains of bleeding gums ster brushing her tet, easy bruising and pin fn her logs whon walking. Her plaslet count is normal, The pathogenesis of her disease is MOST. CLOSELY related to... AL adeliciensy of ATP 1B, cofactor deficiency in collagenase C_ lack of hydroxylation of lysine and prone D. E ppay ‘cofactor deficiency in iysy] oxidase platelet dysfunction Answer: C- the patient has scurvy. Vitamin C hydroxylstes lysine and proline. This isthe binding site for ‘the cross-bridges that strengthen collagen by creating a tiple helix. Henee, in scurvy, collagen is la 11. A 30 year-old man develops an acute onset af confusion, ataxia, nystagmus, and ophthelmoplegis shorfly after the administration ofan intravenous solution containing 5% plucose and normal saline. ‘The pathiogencsis of this patient's neurologic disorder is most elocely relatad to... ‘A. central pontine myetinotysis B. thiamine deficiency ©, Purkinje cell atrophy D. _viralencephalitis E. Budeficiency Answer: B- the symptom complex describes acute Wernicke's encephalopathy. Aleobol excess is the ‘MCC of thiamine deficiency. Thiamine isa cofactor for the pyruvate to weetyl CoA reaction using pyruvate dehydrogenase. Giving the patient glacose in an TV causes the rest of the thiamine to be ‘used up, hence precipitating acute Wemnicke's encephalopathy. Always give IV thiamine before tanging up glucose ‘Optional Genetics questions: material is covered in High Yield 1. A Sl-year-old woman delivers a fall-irm baby that has repeated vomiting of bile stained material. ‘A flat plate of the abdomen reveals air fn the stomach and preximal duodenum and no sie in the remainder of the bowel. The matemal serum q-fetoprotein level is low. The baby has 46 chromosomes. The mechanism of the chil’s disease is most closely associated with.. ‘a Mendelian disorder ‘a Roberteonian translocation ‘nondisunction in meiosis ‘a point mutation of a nucleotide ‘aimicrodeletion disouder PooRp ‘Note: This material is copyrighted. Al rights reserved. (BéwardF. Goljan, M.D) 2002 Answer: B- the patient has Down syndrome based on the low serum a-fetoprotein and signs of duodenal ‘resin, 46 chromosomes indicetes n Robertsonian translocation, where the mother had 45 chromosomes, where the 2 chromosome 21s ate fused into 1 cluomosome. The ehromosoaie from the mother essentially has 2 chromosome 21s + tho 1 chromosome from daddy leads to 46 chromosomes, however, there ae 3 functional chromosome 21. Nondisjunction is the cause of fxisomy 21, 2. Than Aifican American woman with sicklé cell disease bas children with a man lacking the abnotmal f-chain, you would expect. ‘A. 25% of their children to have sickle cell ait 1. 30% of their children to have sickle cel trait ©. 25% oftheir children to have sieke cell disease D. 50% oftheir children to have sickle wel disease E, _all oftheir children to have sickle eel trait ‘Auswer: E~ since the woman is homozygous forthe sickle gene and the man is norma, all the ebildren ‘wall have siokdo wait 3. A I7-yeat-old adolescent presents with primary amenorhea. Physical exam reveals normal ‘secondary female sex charactecisties. Disereie masses are noted in both ingtinal canals, A ‘peculum exam of the vagina indicates a blind pouch. You would expect this patient to also have... ‘Ac aprostate gland B C.anandrogen receptor deficiency D. one Berrbody on a buceal smear 1 ovariesithe inguinal canal ‘Answer: C testicular feminization. This is & SXR (male) disorder with absent androgen receplors. ‘Thesefoc, fetal testosterone cannot stimalate the development of an epididyras, seminal vesicles, ‘or vas deferens from wolfian duct structures. Similarly, dlhydrotestosterone cannot fuse the labia {nto scrotum sad extend the oltoris ino a penis and cannot lead to formation ofa prostate gland. {Left unstimulated by DET, the external genitalia romain feral in appearance. The lower 23rds of the vagina can develon, since it represents the urogenital sinus. All mblierian structures are destroyed by apoptosis, so paticats do not have tubes, ulerus, cervix, or the upper one-third of the ‘agina, henge the blind pouch, 4, While examining « 13-year-old boy during a routine physical examinstion, you. note bilaterally enlarged, non-tender testicles that do not tansilluminate, a high arched palate, and a midhsystolic jection click followed by a short murmur, You ell th school counselor and find that the child as 4 moderately severe attention deficit syndrome. Whick of the following studies would you _scoomsmend on this boy tht would best explain al of the abnormalities noted on the examination? ‘A. Echocardiogram BL Buveal smear . Serum gonadotropins BD. Ieealification of teplet repeat E, _ Chromosome study on his father ‘Answer: D~ patent has fragile X syndrome, Ths isa triplet repeat disorder, henoe the disease gets worse Sn future generations in both the aflected males and the female carriers, Some wenetivsls, therefore call this a sex-linked dominant condition, since female carriers may express the disease. However, other's consider ita sex-inked recessive disorder 5. Prader-Willi and Angelman's syndrome have different clinical features, however they both share @ “defect at th same locaton onthe same chromosome. This is an example of... A. variable expressivity B. a Robertsonian translocation ©. ponctic heterogencity 1D. genomic imprinting "Note: ‘This material is eopytighted. All rights reserved, (Bdward P, Galan, M.D) 2002 B, _ a balanced translocation Answer: D~ mierodeletion syndrome on chromosome 1S, If tho chromoconie ws maternally derived, ‘the patient develops Angclman's syndrome Cheppy puppet” syndrcme). If the chromosome is paternally denved, the patient develops Prader Wall syndcome, 6. Ifthe cartier rate for the sickle cell normality is 1 in 12, the prevelence of sickle cell disease i spproximatsy 1 in a la 288 516 70 1440 3: Sioko ool disease is an AR disease and both parents must cary the abpormal gone, therclore, /12 x V12= 144 black couples are at sk They haves 1/4 risk of having a child with sickle cell disease therefore, 1/44-x 1/4~ 1/576, Hardy-Weinberg equation is he overs of his in that the prevatence is given andthe carrer rate has fo be calculated, To simplify the equation, just ‘work beckwards. For example, /576 = 1M4= {/I44 couples atrisk. 4 1/144= 1/12. 7. _ Whish one of the following transplacental infections is asoviated with sendorineutl hearing los, blindness, nd periventricular ealifications? z jeerk A. Cytomegalovirus B. Toxoplasmosis c D, E. Answer: A~ urine is the best body fluid to culture CMV and to look for intranuclear inclusions in the renal tibularcells 8. A pregnant woman during her first trimester developed fever, a maculopapular rab, arthritis, xd fein postu Inphadenepaiy. Which ofthe flloning complications could potentially ‘oour in her newborn child? A. Periventricularcalification B. Saddle nose deformity ©. Sensorineural hearing loss D. Limb hypoplasia E. _ Craniofacial sbuormalities ‘Answer: C-mother has rubella, Sensorineural hearing lcs isthe most common shnormality. Seda nose deformity refers to congenital syphilis, periventricular calcification to.CMY, limb hypoplasia to ‘thalidomide, and eraniofacial abnorinalities to isoletinoin seid used in treating eystie anne. 1. In whieh of the following sites is the MOST COMMON primary cancer an adenooarcinoma? SELECT 2 ‘A. Esophagus B. Bladder Larynx dD. Cenix FE Lung Answers: A, B: adenocarcinoma of the distal esophagus related to Barretts esophagus has replaced ‘squamous cancer of the mid-esophagus as the MC cancer of the esophagus, Adenocarcinoma has also replaced squamous cell cancer as the MC primary lung cancer. 2, Which of the following genes regulates Kinases in the cell di ‘important role in the development of human canoer? 9 sion cycle, hence assuming an ‘Note: This material is copyrighted, All rights eserved. (Edward F. Golan, M.D}) 2002 eas proto-oncogene xb B proto-oncogene Rb-l auppressor gene esmye proto-oncogene . p53 suppressor gene B_ p-53 produces product that inhibits active oyslin D-depencient kinase, which normally ‘hosphorylates the Rb protcin and allows the cell to enter the $ phase, This allows the eel time to repair defeets in DNA before allowing the eell info the $ phase, Cells that sre damaged beyond repair are removed by apoptosis. 3, A.primary cancer is more common than metastasis in which of the following sites? A. Lymph node B. Colon, © Brain D. Bone % Answer: Ball the other sites sted have metastasis ore common than primary eater. 4, -A55:yeer-old woman with breast eancer as an infiltrating dota careinoma that is 2 em in size, BRA and PRA postive, metastatic to 5 out. of 20 axilay Tymph nodes, and metas to both the ‘ecebal column.and liver. Whish ofthe following MOST in aences theultimate prognosis i this alien? AL Hlerage 3. BRAMPRA stats C Sizcofthe tamer D. Axillary node involvement Bone and iver involvement ‘Answer: E- extranodal metastasis is worse than nodal metastasis. Remember the TNM staging system, ‘which progresses from the leet to most important prognosis factor in cancer. 5. Which ofthe following isthe second most common cancer and cancer ier n wen nd wosten? A. Malignant melanoma B. Longeaneet ©. Colorectal cancer D. Malignant Iymphoma E. _ Stomach eancer ‘Ansrer: C- since both men ond womien have colons, it should be no supriss tha eal cancers second ‘in incdenee end mortality in both men and women 6 A tumor dit could potently produce Cashing’ syndrome and hyponetremi is most likely Aocate inthe. ‘idney placeata liver tiyroia Jung Answer: B~ small cll cancers of th hing produce ACTH and ADH 7. Both AFP and (ACG sre mos likely to be elevated in which primary tumor site? Fope> A Lung BL Testicle © Liver D. Colon E Ovary ‘Answer: B- these tumor markers are always ordered in testicular cancer. AFP comes from yolk sae tumors (endodermal sinus tumors), while f-hCG comes from choriocarcinoma. 10 Note: This material is copyrighted, All rights reserved, (Edward F, Golan, MD) 2002 8. Which ofthe following cancers is prevented by immunization with a commonly used vaccine? ‘A. Pasiroati earcinoma B. Stomach carcinoma ©. Transitional cell exreinoma D. Hepatocellular carcinoma EB. Cervical carcinoma Answer: D~ pation“ will not get HBV, HV, or hepatocellular carcinoma due to HBV 1. A.19:yeancld Afticen American woman presents with fatigue and exercise intolerance. She fas a history of menoctagia. A CBC reveals a mild microcytic anemia, w low normal WBC count, and 3 ‘normal platelet count, A comecied reticulocyte count is <2%, Which of the following sets of laboratory data best represents the hematologic findings inthis patient? Serum Iron TIBC % Saturation RBC Count Ferritin A. normal oral normal high normal Blow low low low high tow high Jow low low D. high low igh. nnocmal high B.uorml nnonmal normal low: normal Amswer: C menonhagie gives away the answer a8 a cause of ion deficiency in young women, A = ‘thalassemia note the normal iron studies end high RBC count, B= ACD, D= iron overload~note ‘he ow TIBC (teva that excess iron stores decreases transferrin synthesis), ~ acute blood loss initally, the RBC count, Hab end Het are norm, since whole blood is fost. Plasma is replaced Yofore RBCs, so the REG deficit is eventually uncovered. Giving the patient isotonic. saline immediately uncovers the RBC deficit and the High, Hc, and RIC count will be low. 2. Which of the following characterizes pemicious anemia rather than Bre deficiency from other causes? ‘A. Increased plasms homocysteine lovels B. Increased urine metiylmatenie acid ©. Decreased vibratory sensation D._ Hypersegmented neutrophils E._ High sorom gastrin levels Answer: Bother differences include (Geason for high gastrin loves), aujoanibodics against {ntrnsi fhotor and parietal cell, correction of Schullngs test wilh intinsie factor. 3. A 2seyear-old man has a motonyele accent outside the emergency room. Physical exam in the ‘erence room reveals a weak, thready pulse, cold, clammy skin, and a blood pressure of 60/40 sm Hg, An open right femoral fracture is present as well as tedemess over the left lower ribs. Which of the following would you expeat in this patent if blood sample was dravm prior 10 insertion of an IV line with 0.9% rotmal saline? SELECT 3 ‘A. Normal hemoglobin and hematocrit BL Notmal effective arterial blood volume C.__ Ineredsed central venous hydeosatc pressure 1D. Decreased mixed venous oxygen content E. _ Decreased pulmonary capillary wedge presse ‘Answers: A, D, B= the explanation for A isin question 1. MVO2 is deereased because the patient isin Ihypovolemic shock and the tissue cam extract more oxygen from the blood. The LVEDV is decreased because plasma volume is decreased. 4. A -year-old African American child with side cell anemia peosents with a high over and michal rigidity. Scleralicterus and hepstosplenomegaly are present, A CISC reveals a moderately severe ‘nonmocytie anemia and 2 WBC count of 21,000 cell. with left shift, A ucinalysis fs normal. A " ‘Note: This matesial is copyrighted. Allights reserved. (Edward F, Goljan, M.D, 2002 spinal tp revels turbidity, ineeased protein, nd neutrophils. You would expest a gram stan of SF an blood culture to reveaL.. 7am positive dplocooct ram negative diplococet grat postive scsi ‘gam positive rods ‘ram negative coceobaili Answer: A ~ Streptococcus pneumoniae sepsis is the MC cause of death in children with sickle eel diseate. Hence, the importance of Pneumovex and prophylactic antbitice. 5. A febrile 23-year-old college coed presen's with fatigue and difficulty with swallowing. Physics! ‘exam reveals exudative tonsils, palatal peteohia, cervical lymphadenopathy, and. tender Iepatosplenomegaly. A CBC revesls a mild microeytic anes, ImphucyiceFeukoeosis with ~ 20% ofthe lymphocytes exhibiting apical fates, and a mild thrombooytopenia. You would spect this patient 19 have Ay alowTIBC 1B. anormal serum fein Camelovated total biiabin D. _heterophile antibodies E _ normal serum AST and ALT titers Answer: D~ the patient ha ifeetious mononucleosis ‘6A 6Syearald man with an elevated REC count has a hiblory of gout, frequent headaches, and rites ner bathing. le presents with a sudden onset of abdominal pan and bloody diaries. rooe> RBCMass Plasma Volume Sa0z erythropoietin A. Tnoreased Normal Decreased Increased 3B. Tnoreased Normal Normal Increased Increased Tnereased Normal Low D. Noumal Decreased Normal Normal Sa02~ oxy saturation _Anawer: C- the patient has PRY. The othr anives ate: A COPD o€ eyanatic CHD, B = ectopic BPO ‘production (eg. renal adenocarcinoma, squamous cancer of mg), 1 = relative polycythemia (volume Sepleted) 7. A 2-yearold woman presens with fever, scleral oterus, and erampy right upper quadrant pain. Physical exam revels fendersss to palpation inthe right upper quadrant mn mild splenomegaly ‘A heroogram exhibits a total leukoeyte count of 23,000/mn, hemoglobin of 10.0 gL (12.0-16.0 BAL), 2 corrected reticulocyte count of 6% (0.5—1.5%), a mean corpuscular volume (MCV) of 80 jun? (80-100 yum’), and 2 mean corpuscular hemoglobin concentration (MCHC) of 38% (31-36% Hblect) The total bilirubin is 32 mg/d. (0-1~1.0 mg/dL) with a direct (conjugated) faction of 0.4 rme/Al. (00-0.3 mg/l). Serum alanine aminotransferase is 20 UM. (8-20 UML). 4m ultrasound reveals numerous siones in the gallbladder, The common bile duet isnot dilated. Test on these findings, which ofthe apply to this case? SELECT 3 ‘ncreased RBC osmotic fragility Caleium bliubinate stones “Low serum feritin RBCs with «defect Positive direct Coombs! ‘Decreased unconjugate bilirubin Answers: A, B, D- the patient has congenital spherocytosis. Increased extravascular hemolysis increases ‘he srount of blicubin i bile leading to calcium bikubinat stones and cholecystitis mrpoEP 2 ‘Note: This material is copyrighted. Al rights reserved. (Edward F. Goljan, M.D.) 2002 8 A significant number of employees in a pottery factor develop abdominal colic and diarriea. In addition, a few of the employees complain of buming feet and muscle weakness in the upper and lower extremities. Based on these findings, which of the following are expected laboratory findings in these patients? SELECT 4 "Normocytic anemia Coarse basophilic tipping Low serum ferritin Increased blood Pb levels, Increased RBC protoporphyrin levels Tnoreasod plasma homocysteine levels. G, Ringed siderobiasts Hecom> Answers: B, D, E, G- they have Po poisoning with pecipberal neuropathy and the abdominal clic. The ‘pint tsa i pottery oflen contains Pb. Other seenarios-used on boards include eating old paint, ‘working in-an automobile faetory (incinerate bateies, drinking moonshine (lead lined radiators). ‘A.33-yearold Affiean-American medical missionary, who recently retumed ftom 2 2 yesr tour in “India, i diagnosed with leprosy. Aer 1 week of therapy, he develops fever, chil, love back pain, ‘and dark colored rine. A CBC reveals 2 hemoglobin of 6 gL. (13.5-175 g/d), leukocyte count of 15,000/mm? (4500-11,000/mm’), and a platelet count of 450,000/mm’ (150,000—400,000m) ‘A corrected reticulocyte count is 18% (0.5-1.52). A direct and indirect Coombs! testis negative. ‘The peripheral smear exhibits polychromasia and RBCs missing parts of thair membrane, There is & “positive urine dipstick for blood. The urine sediment i reported es normal. Which ofthe following statements apply to this case? SELECT 3 ‘A. Hoinz bodies are likely present 1B. Predominantly extravascular hemolysis © LowRBC glutathione levels D. Palit is taking dapsone E, Autosomal recessive diseave Answers: A, C, D~ patient has GOPD deficiency (SXR disease) with hemolysis secondary to dapsone. 10, During acute temolysis enzyme assays are usually normal, since only tbe eells conning the ‘syne remain behind, while those witht the enayne ae hemolyed. ‘An afebrile 65-year-old man presents with fatigue and sbstermal chest pain with exertion thats telioved by resting. Physical exam revealx a blood pressure of 100/70 mm Hg, pulse of 110 ‘ats/minut (diminished amplitude), pale conjunctiva, end a harsh grade I-IV systolic murmux ‘with dation into the carotid arteries, The intensity ofthe murmur iereass with exptation and ‘when the patient is ing down. A CBC exhibits moderately severe anomie with © mean corpuscalar volume (MCV) of 16 pu (80-100 jm), Nemoglobin of 7.5 pL. (1:5-17.5 gL), oe count of 8,000/niri” (4500-11,000/mm"), and a platelet count of $00,000/mm* (150,000-400,000/mm) "The peripheral smear uncovers mumerous fagmented RBCs. The corrected reticulocyte count is 159% (0.5-1.5%). There is a positive urine dipstick for blood but "Garena reset nthe einen. Whi fh lowing nyo sce ELBE 4 Positive direct Coombs test Lowen kpogots CC. Sohistooytes D. Hemoglobinuria E. Aortic regurgitation F, _Low serum ferritin ‘Answers: B, C, D, F— the patient has aortic stenosis and a macrosniopalhic intravascular hemolytic ncn with chistoytes, Loss of Hg in the urine has caused irom deficiency. Thombocytess Common in conic ron defen. B "Note: This material is copyrighted. AMI rights reserved. (Edward R, Golan, M.D.) 2002 11, An asymptomatic, normotensive 21-year-old AGican-American woman is noted to have ‘microscopic hematuria on an otherwise normal physical exam. The urine cultae retams negative, ‘Acrenal ultrasound is reported as normal, Her bemogiobin concentration is 12.8 g/L (12.0-16.0 {g/L) and the peripheral smear is reported as nonwal. Based on these findings, which of the following is the next best step:in the management of this patient? ‘A. Sickle cell preparation 1B. Reticuloeyte eount C. Renal biopsy D. Cystosopy F. No further work-up ‘Answer: A- the patient has sickle cell trait with miroinfirotions in the renal medulla. O; tension is low ‘enough in the medulla to induce sildimg. 12, A pregnant 21-year-old African-American woman lus a mild microeytic anemia with « normal [RDW and slightly elevated RBC count, A Hgb electrophoresis is reported to be abnormal. You suspect thal the High electrophoresis showed... SELECT 3 ‘A. increased High S| 1B, decreased Hgb A CC. incteased High As ‘D. increased Heb F E. increased Figh ‘Answers: B, C, D~ the patient has B-tbalassemia minor, Sines P-chain symthesis is decreased, c-chains ‘an combine with &-chns to produce Higby and combine with y-chains to produce HgbF. HighA fs deereased due to the lack of P-ehains. 13, A 46yeerold man with diastolic hypertension develops fever, jaundice, anda severe hemolytic tmemin. while taking methyldopa, ‘The hematocrit is 159% (41-839) and the uncomected reticulocyte oount ia 24%. Spherooytes and polyehromasia are present in he peripheral smear ‘Which ofthe following most key apply to this case? SELECT 3 A. Intravascular hemolytic anemia B. Reticuloeyte index is 4% ©. _Unoonjugated hyperbiliubinenia D. ‘Type hypersensitivity reaction BE. Autoantibodies against Rh antigens Answers: B,C, E-autoimmine bomolyic esemia. Most eoect for the anmia and the plychromasa ‘Type lt hyperseostivity reaction. 14, N65 year-old man prevents wit fever and chills, Physical examination reveals generalized, non tender lymphadenopathy, hcpatosplenomegsly, and scattered potchia and ecchymoses over the nlerior chest A CBC report indicates a hemoglobin of 8.2 gd (135-175 gl), leakooyte count ‘of 70,000/mex(4,500-11,000/mm") with 90% mature and irmature appearing lymphocytes, and 2 platelet count of 80,000/nax* (150,000-400,000/mm’). The peripheral smear contains numerous ‘smudge evils. A blood culture is positive for Strepracoccus preumaniae. The total serum protein concentration ie 4.0 g/L. (60-78 wil). A ceruo poten electrophoresis exhibits flat y-globulin peskand deereased alburin. Which ofthe following eppy to this ease? SELECT 3 ‘A. Leukemoid reaction secondary to sepsis 1B. THypogsrumsplobulinemia ©. Chronie lymphooyti lukeria 1D. Low leukoeyte alkaline phosphatase score B. Beellmalignaney “Answers: B, C, I~ the patient has CLL with hypopammaglabutinemia. Iti the MC leukemia and cause ‘of generalized lymphadenopathy after 60 ys of age “4 ‘Note: This material is copyrighted. AI rights reserved. (Edward F, Goljan, M:D.) 200% 1s ‘A Ayear-old boy prosents with fever, epistaxis, and testicular pain. Physical exam reveals ‘gencralized, non-tender lymphadenopathy, hepatosplenomegaly, stemal tenderness to percussion, ‘and widespread petechia and ecchyrases. Both testicles art enlarged, slightly tender, and do not ‘ransilluminste. The CBC report indicates a hemoglobin of 6 g/dL. (11.0-14.0 w/a), leukocyte count of 30,000/mm? (5,500-15,500/mi?), and 2 platelet count of 0,000/m? {(150,000-400,000/mm?). A bone marrow aspirate reveals sheels of cells shilsr to those present in the peripheral blood. Which ofthe following apply to this case? SELECT 3. ‘A. Cells aromost ikely CALLA positive B. Cells are most Hkely Tat positive ‘C. Acute lymphoblastic leukemia D. Lymphoid leukemoid reaction 1. _Louleeria derives from trilinege myeloid stem cell, “Answers: A, B,C: the patient has acute lymphoblastic leukemia 16. ‘A AS year-old man presen with fever, weightloss, and sweating. Physical exam reveals = ormotensive individual with generalized, sontendet lymphndenopathy and massive hepatosplenomegaly, The CBC. repoet indicates hemoglobin conbentation of 7 gilL (150,000-400,000imn"), leukocyte eouut of 150,000imm? (2,500-11,000/mm?), and a platelet unt of 650,000? (150,000-400, 000mm), Th periphers amet demonstrates neutrophils at all tages of development (1% myeloblasis), mature and smmature eosinophils and basophils, and thrombooytosis wih giant platelets, A bone marrow aspirate ia dry, however, the bone matrow biopry reveals «hypereslalar marrow wilh an increase in zetiouln bers. prints ofthe biopsy reveal «similar differential count as that oberved athe pecpberal ood, Which ofthe following ‘uional laboratory test abormalites would you expect inthis patient? SELECT 3 ‘Low leukooyteallaline phoophatase score Positive trae resistant acid phosphatase stain ‘Aver sods in myeloblasts Positive Philadephia chromasome study Positive ber-fusion gene study {514 translocation mapOR> Auswers: A, D, E- chronic myelogenous leukemia, Blasts do not contain Auer sods (only acute 17 ‘myelogenous leukemia has bists with Auer rods) 40 19-year-old Affican American woman presents with fatigue and exercise intolerwace. She has @ history of meniomthagia and sporadically takes ferrous sulfate tablets. A CBC reveals a mild ormocytie anemia, a low normal WBC count, a normal platelet count, and normal RBC ‘morphology. A corrected reticulocyte count is <2%. The next most important step i a. A. serum fertin B. Coombs! test C. serum folate’, D. Hb electrophoresis sickle cell preparation “Answer: A~ early stage of iron deficiency. Sick cell trait does not have anemia, Recall that all the ion 18, studies (iron, TIBC, % saturation, ferritin) are abnormal before there is any anemia, The anemia is first normoeytic and then becomes microcytic. ‘An afebrile 80-year-old man with the myelodysplastic syndrome is symptomatic with a 7 g/dl. hemoglobin, He is given 3 units of packed RBCs and on the following day hus a hemoglobin of 8 ‘env. His diceot Coombe" testis negative, A dipstick of usin for blood is negative. The MOST LIKELY cause for only a 1 gm/dl. rise in the post-transfasion hemoglobin concentration is. ‘A. destruction of the RECS in the bone marrow B. _amicroangiopathie hemolytio anemia © anantoimmune hemolytic anemia 1D. desiruction of the RBCs in spleen 1s [Notes This material is copyrighted. All ights reserved. (Edward F. Goljan, MD) 2002 Ea gastrointestinal bleed Answer: E- think most common lack of response to transfusion is most commonly due to & GI bleed. ‘TheHgb should increase by 1 gavl. for every unitof packed RBCs and the Hit should increase by 3%. 19, A 22yearold college student presents with petechia, cechymoses, epistaxis, generalized lymphadenopathy, and hepatogplonomegsly. A CBC reveals 2 normooytie anemia, 'a total WBC ‘count of 30,000 cels/ul, and thrombocytopenia. Abnormal “blast ces” are noted in the ‘smear. The PT and PTT are prolonged, and the D-dimer testis postive. Which of the followin apply to this case? SELECT 2 A. Positive tartrteresistant acid phosphatase stain B. ‘Therapeutic response to transetinaie acid © Tow leskooyte alkaline phosphatase seore D. (15:17) translocation B. _Tnvasion of the gums ‘Answers: 3, D- patient has ncvte progranulocytic leukemis with DIC, A positive TRAP stain is hairy ‘ell leukemia and invasion of the gums is acute monocytic leukemia, A low LAP is chronic granulocytic luli. 20, Which of the following is more often assosated with Puthalassemia minor than iron deficiency? SELECT2 A LowMcy B. creased RDW ©. Inreased serum fritin D. Normal ta high RBC count FE. _Abnonmal High electrophoresis Answers: D, E— remember that iron staies are normal in mild thalassemia (oor i). Tnereased RDW is characteristic of iron-deficiency, Both have a low MY. 21. A 28-year-old woman presents with fever, fatigue, and scleral iotenus. She is not taking any prestsiption or ovecthe-connier medications. Physical exam reveals generalized painfil Iymphadenopathy, hepatosplenomegaly, and 2 facial rash in a buterfly distribution. A CBC ‘cahibits 2 Heb of 6 gL, sligtly increased MCV, thrombocytopenia, and slightly low WBC ‘count. The peripheral smear demonstrates spheroeytes, REC —polychromati, and occasional ‘mucleated RBCs. No hypersegmented neutrophils are present. The uncorrected reticulocyte counts 15%, Which of te following apply to this case? A. Bilirubin in the urine B. Positive ditect Coosbe'test Unconjugated hyperblinbinemia D. _ Reticulooyte index < 296 E, Positive serum antinuclear antibody test ‘Answers: B, C, E- patient with SLE and a warm AILTA, which is an extravascular hemolysis leading to smincteasc in UCB and jaundice. UCB cannot be fitered in the urine like conjugated bilirubin Tae reticulocyte inden =4.5% 22. You would expecta mature RBC to... SELECT 2 A. utilize lactate deiydrogenase B. utilize glucose 6-phoephatase © convert glucose into glycogen D. utilize pyruvate dehyerogenase B, _haveanet gxin of @) ATP and (0) NADA “Answers: A, E~ they have no mitochondria and use anacrobic glycolysis for energy 23. -An aftrile patient with SLE has a mild normocytic anemia with an elevated Jeukooyte count ‘exhibiting nevtrophilic leukocytosis, lymphopenia, cosinopenia, and a normal platelet count. There 6 ‘Note: This material is copyrighted. All rights reserved. (Edward F. Gajan, M.D) 2002 {is no left shift or toxio granulation present in the smear. Her stool gusiac is postive. The ‘hematologic findings are MOST CONSISTENT with... ‘A. acute leukemia B. aviralinfection C._abactrial infection D. grumnegative sepsis E, _ corticosteroid therapy Answer: E~ deoresse adhesion molecule synthesis and destroy lymphocytes and eosinophils 24. Which of the following hemolytic anemias are primarily due to extravascular removal of RBCs?” SELECT2 ‘A. Mioroangiopathic hemolytio anemia B. Warmauloimmune hemolytic anciia Congenital spheroeyins GEPD deficienoy ‘Paroxysmal poctimal hemoglobinuria F, Cold auloimmune hemolytic anemia Answers: B, C; NOTE the athes are primarily inerevascular hemolysis 25. Which ofthe following conectly describe anemia? SELECT 2 A. Hypoxemia J. Lowoxygen saturation Decreased oxygen content D. Laftshifted oxygen dissociation curve Tissue hypoxia Answers: ©, B~ only the Hab concentration is decrease, Gas exchange is normal, so the PaO, and SO» ‘shouldbe normal. 26, -A.65-year-old man presents with joint pains, palpable porpure, and henstosplenomeyaly. There is xo evidence of lymphadenopathy, A CDC exhibits a normocytic anemia, thrombocytopenia, and neutropenia. A few “blast cell” with inegular eytoplasinis borders aré noted inthe peripheral blood. A bone marrow biopsy reveals a monomorphic inflate of eels with abundant cytoplasm having “fred egg” appearance. A special stain is pending. Which ofthe following apply to this ease? SELECT 2 A. 1922) translocation B. Epstein-Barr virus assoiation € Beeltmalignmey D. Positive tartrate resistant acid phosphatase stain E, Low leukocyte alaline phosphatase score ‘Answers: C,1D~the paticnt has ity cell leukemia 27, ‘An 82-year-old man has along history ofa severe slighlly macrooytic anemia and pancytopenia. He ‘oquites a transfson of packed RBCs every 2 weeks in order to miantan his Hb concentration around 10 pil. His peripheral smear consistently shows a dimorphic RBC population associated with oceasional myelobists and progranulocytes. A bone marrow aspirate reveals numerous ringed ‘sideroblass and a 10% myetobiast count. Ths patient MOST LIKELY has. ‘A. aonte myelogenous leukemia 1B. agnogenic myeloid metaplasia C. the myelodysplastic syndrome 1D. chronie myelogenous leukemia F. _aneutrophilc leukemoid reaction Answer: C- these patients frequently progress into an acute myelogenous leukemia, Small and large [RBCs and blsts are common in the peripheral blood. Ringed sideroblats are also a common feature inthe bone martow, 7 ‘Note: This material is copyrighted, All tights reserved. (Edward F, Goljan, M.D.) 2002 28, A'SS-yeacold woman with an sboormal mammogzan ofthe lft beat, palpable left axillary lymph ‘odes, and law beck pun is noted to have a mid nomosyte anemia, thrombocytoss, a elighily tlevated WAC cout, and 2 peiphen! smear denoustating nucleated RBCs, and occasional ormuoeys ad melee he ing ate onsen: wh. SELECT 3 hronio myelogenous leukemia 3 Senempcogeoa eens ©. mmarow inflrative disease D. loukoerythoblasie smear E. _lenkemoid rection Auswers: G, D leokoerythroblastic smeer from metastasis to bone. Tumor is pushing normal macrow ‘hematopoietic cells int the peripheral blood. 29. Hosinophili s commonly associated with... ELECT 4 ‘A. pinworms acute appendicitis, (Crohn's disease neute diverticulitis acute cholecystitis acute mygearial infarction sheuratoid ardiritis ‘Answers: A, C, D, E- the Gust 3 are bacterial infections due to E. coi. Catesholamines and tissue 1 neutrophilic leukocytosis in an AMI E, autoimmune hemolytic anemia Answers: A, C— parvovirus also produces splastic anemia in patients with an underlying hemolytic anemia (¢§, sickle cell disease, congenital spheroeytosis), 32, A2yrold child bas a microeylic anemia. Which of the Zollowing isthe first step in the work-up of the patient? AL Serum feritin B. Stool guaiae © Bone marrow D. Direct Coombs test BE. Hgb electrophoresis Answer: B- most mierocytic ancmias in children are due toa bleeding, Meckel’s diverticulum 33. A.55-yrold man has a microoytic anemia. Which ofthe following isthe first step in the work-up of 18 ‘Note: This material is copyrighted, Al rights reserved, (Edward F, Goljan, M.D) 2002 ©. Bone marrow D. Dieet Coombs test B. _ Hyb electrophoresis Aaiswers most microcytie nnemias in adults over 50 are due to eolon cancer 34, In lead poisoning, the encephalopathy is directly attributed to. ‘A. shincrease in RRC protoporphyrin B, saninerease in 5-aminolevulinic acid ©. Pb depositing in the brain D. _amalterstion in Starting’ forees ‘E, tissue hypoxia secondary to anemia aos 3s, noapen vt pemeshilyeeing oer oes Ina paticat with Bis deficiency who is being treated with pharmacologic doses of folate, which of ‘the following will be corrected? A. Megaloblastc anemia Glossiis B. . Pancytopenia ‘D. Vibratory sensation in the lower extremities E, _Propeioception Dementia Answers: A,'B, C NOTE: the neurologic deficits remain, henee the imporiance of making the correct diagnosis 36, Newborns with sickle cell disease do not have hemolytic or vasoocchusive erises at bitth sinee.,, SELECT 2 high concentration of Hgb F inhibits sickting B._tevels of igh S are not high enough o induce siding © Hgb A inbibits sickling D. splenic macrophages are of insutficint number to remove sickle cells E. _ the spleen uaps the siekle cell Answers: A, B-Hgb A is never present in sickle ell disease 32, A 19sy-old Affiean American man with siekle cel disease develops bone pain in the femur. A radionuclide bone scan reveals a Iytic lesion in the metaphysis of the femur. The pathogen most likely responsible forthe pationts bone disorder is A. Staphylococcus aureus B. Strepiococcus pnewmoniae © Salmonella parasyphi D. Prewdomonas aeruginasa E, Hemophlus influenzae Answer: C~in chien without sicko cell disease, Staphylococeus aureus is the MCC of osteomyelitis, not Salmonella 38. Inapatient with sick cll trait, you would expect the MSH endonuclease studies to revel. 1.35 kb segment 1,15 kb fragment 0.2 kb fragment A. @)segments ‘none ‘none Be (U)segment (2) fragment (1) fragment c none (@) fragment @) fragments Answer: B: the abnormal chromosome has (I) 1.35 kb segment, while the normal chromosome is cleaved into (1) 115 Kb frapmeat and (1) 0.2 kb fragment, Patient A tas sickle cell disease, with (2) tncleaved 1.25 kb segments. patient Cis normal, and both 1.35 kb segments have heen cleaved into (2) 1.15 Rb fragments and (2) 0.2 Eb fragments ‘ ‘Note: This material is copyrighted, All ights reserved, (Bdward F, Goljan, M.D.) 2002 ‘Lymphoproliferative questions “ems 1-2 ‘A G5-year-old woman presents with fatigue uid pain net lower back and eb, She states that her urine flow has also decreased cramaticaly in ths last few days. Physica. examination reveals Hema and vertebral percussion tendemess and bilattal eonjuncival pallor. A CBC exhibits extensive rouleaux, 2 hemoglobin of 7.5 g/L (120-16.0 w/a), & total leukooyie count of 4,300/mn? (4500-11,000/mm?), and a platelet count of 125,000/mn (150,000—400,000/mm”). A tinalysis exhibits a 2+ dipstick for protein and 4 precipitation using sulfsalcytic aid (SSA). Renal tubular casts are noted inthe sediment. A chest x-ray reveals generalized osteopenia in the ribs and vericbra as well as ltl Iyti lesions in tho ribs. 1. Based on the patient's history and preliminary laboratory findings, which of the following addins abnormalities would you expect? SELECT 3 A. Byperealeemia B. Normal erythrocyte sedimentation rato % © E._IeM monoclonal spike in serum D. creased light chains inthe urine Y E, Malignant plasma celistn a bone marrow aspirate “Answers: A, D, E: the patent bas ‘ote the usin findings that sereenfor BY protein Ms akeadyoosures 2. What additonal tests should hg performed to confirm the diagnosis? SELECT 2 ‘Bone marrow aspirate / Renal biopsy ‘Radionutide bone soan Intravenous pyctogrms (VE) 7 Se; 3c immuncelectrophoresis Answers: A, E: ‘would further exacerbate the patient's renal disease. IEP identifies the abnormal ‘Tg and light a. SS ems 3-7 FOOBP Histocytosis X Sezaty syndrome Hodgkin's disease ‘Burktt’s lymphoma ‘Smmunoblastic lymphoma ‘Waldenstrom's macroplobalinemia 3. A.S-year-old piel presents with calidky pain secondary to entrapment of small bowel by enlarged para-aortic lymph nodes. A section of lymph node removed at laparotomy reveals a diffuse ‘neoplastic infilteate of small, round lymphocytes with a "stay sky" appearance. Answer: D: B cell malignancy with a (8:14) of the e-mye proto-oncogene 4. A 68-year-old. man presents with multiple plaguelike lesions on his skin, generalized |ymphadenopethy, and hepatocplenomegaly. A biopsy reveals atypical lymphooytes inflating the ‘epidermis. Neoplastic CD, positive lymphocytes with prominent nuclear clefts are noted in his peripheral blood. Answer: B- Sezary syndrome. Its called mycosis fungoides ifthe cells are notin the blood. 5. Add-year-old child prescnts with exophthalmos, polyuria, and mulls sons in the sal. A ‘bone aspirate reveals an infiltrate af neoplastic cells that Answer: « ‘Hand Schller-Christin disease. CDI) a histiocyte marke SS 0 =RpOEP "Note: This material is copyrighted All rights reserved. (Edward F. Goljan, M.D.) 2002 6 A @0-yearold man has a normocytic anemia, an elevated erythrocyte sedimentation rate, scnctlized lymphadcnopstiy, hepatosplenomegly, hyperviscosity syndrome and an aboormal finding ona serum protein electrophoresis. Answer: F- Waldenstonts macroglobulinemis~ an TeM monoclonal spike must have been present, ‘Multiple myeloma doesnot usually have an lM spike. 7. -A283yrold woman has an anterior mediastinal mass and non-ender lymphadenopathy inthe ight supeselavinlae node ‘Answer: C: nodular sclerosing Hodgkin's disease. This isthe classic presentation of nodular sclerosing HD in women or men, 8. Restrictive cardiomyopathy, macroglossis, end nephrotic syndrome are all associ. with x disorder characterized by. AL nvoplastc histiocytes ina bone mao aspirate 5B. nooplhste plasms oes fa hone meow aspirate , neoplastic Iymphoostes witht) translcaton D. _Iyaphoplasmaeytoid cells and a monoclonal increase in eM E. _ material exhibiting apple grea birefFingence under polarized Tight ‘ossrer B these ae al features of amyloidosis ‘Hemostasis questions L.A 22-year-old Afiican American’ woman with no previous bleeding history develops persistent ‘leading of her gums and tooth socket following dental surgery. Additional history reveals problems related to heavy menses that significantly resolve when she is taking birth eonteo pill snd recur tothe same level of severity when she discontinues the medication, Hematalogie stusies reveal de following: pactial omboplastin time (PTT) 55 see (2840 see), prothrombin time (PT) 12 seo (11-15 seo), factor VII: caagulant 20% (50-150%), factor Vl angen 40% (60-150%0, ‘lceding time 15 min (27 min), platelet count 300,000 mn (150,000~400,000 mm), hemoglobin 13,0 fll (12.0-1600 g/L). Which of the following laboratory tests is most sefil i confining, the cause of her bleeding? ‘A. Leukocyte count B. Ristoetin cofactor assay C. Hemoglobin electrophoresis D. Sickle cell preparation E,_ Serum ferritin Answer: B~the patient has von Willebrancts discise 2. A.23-year-old man requires root eanal for an abseessed tooth for whieh he has en taking pain ‘medication. On the day prior tothe procedure, he develops a severe nosebleed, which prompts his ‘dentist o order a few laboratory studies, which areas follows. PTT 35 see (28-40 seo), PT 13 ace (11-15 960), bleeding time 16 min (27 min), platelet count 200,000 ra? (150,000-£00,000 max), Hab 15.5 wAL (13.5175 pill) Which of the following Kemostasischnormalites is mos likely present in this patient? ‘A. Von Willebrand's disease Hemophilia A ©. Factor IX deficiency D. Acquired platelet defeet E. Acquired vascular defect Answer: D- the patient is most likely taking aspirin or other type of NSAID. The only Rx that could correct this problem would be a platelet transfusion, which is erainly not indicated in this case but ‘would bein a ife-threatening bleed. a ‘Note: This material is copyrighted. Allxights reserved, (Edwatd . Goljan, MID.) 2002 Ttems:3-4 ‘A.G3-year-old man with urinary retention seepndery to benign prostatic hyperplasia develops fever and chills shortly after insertion of an indwetling catheter. Physical exam reveals warm skin and a ‘bounding pulse, Within 24 hours, he begins oozing blood out of venipuncture sites and from his _mucous membranes. Bechymoses appeer over his trie nd extremities is urine output decreases to <400 mL/day. Laboratory studies reveal the following: Hgb 10 g/dL (13.5-17.5 wdL), WBC ‘count 2,000/mm? (4,500-11,000/mn?), platelet count 140,000/mer? (150,000-400,000/nn?), PTT 42, see Q8-A0 seo), PE 18 see (1-15 seo), plasms, fibrinogen 150 my. (200-400 m/l.) Tbcn(open) degradation products >10 yglmaL (10 jg), Dimers positive (negative), Blood ‘alles pending, senim BUN 80 mg/l. (718 mg), orum creatinine 8 mA. (0.6-1.2 mela) ‘Which of the following apply to this case? SELECT 4 “Thrombotic thrombocytopenic purpura isterinatedinravasculer coagulation arSger> Fe ._ Badotoxie hock Answers: 3B, CE, H: the patient has DIC snd isin real flue (BUNIcreatinine atio<15) 4. "Which of he following is the most effective treatment for this patient? A. Fees frozon plastea B. Packed red blood cells ©. Phaziecconcentates D. Lov dove beatin . Anubiotios E. _ Cryoprecipitate “Answer: : in DIC, the most effective Rx is o Rx the cause of the DIC, in this ease, endotoxie shock secondary to £. cot, The blood components ar also indicated, butecsentally feed the fireand Keep the patent alive until the underlying disease s eradicated. 5. Which of the following laboratory test results are more often associated ith classical von Willerana’s disease rater than wild hemophilia A? SELECT ‘Normal prothrombin time ow fctor Vticoagalanactiviy “Abnormal rstosetin cafctor assay x § i ‘Low Vill: antigen ‘Nommal platelet coust ‘Response to desmopressin acetate Answers: C, E, good comparison question Hema 6-12 Platelet count Bleeding time PTT vr ‘A. Decreased Prolonged Normal_~—Normal B. Decreased Prolonged Prolonged Prolonged © Normal Prolonged Prolonged ‘Normal D. Normal Normal Prolonged — Nowa Normal ‘Normal Prolonged Prolonged E Normal Normal Normal Prolonged G._—_ Normal Prolonged Normal ‘Normal 6, A-yr-old child has eaten rat poison and is hemorrhaging 2 Come EK i AGinsig, ‘Note: This material is copyrighted. All rights reserved. (Edward F. Goljan, MD) 00, Ute Answer: E-rat poison is warfarin, which blocks all thq(tamin K-dependent fac PT and PTT. | are prolonged: frst on eprint erent dep wou Bonbon heparin eienees ATIL, which nels most inne ftetors and moet of the ial eznaon patsy factors hence bot PT aad PIT re poling 8. A 28-yrold man has been envenomated by a rattlesnake and is bleeding from all orifices and all ead sees da pail ns DIC ai comming platelet, nog, potuonbi,V, VI raw hamburgers and now has a hemolytic anemia and renal faikwe | due: to°0157: H7 serotype of E. coli. Remember that only platelets are consumed, not we Dic eee 10, MME Tamily history of a bicesting disorder which began with his mother’s father Answer: D; hemophilia A, maternal father tunsmits the disease to all his daughter's (SXR trait) who are asymptomatic trier The dager the pee to Sth ath sons LL, At able -yearold boy presents wk estate I werk aller en upper sespizaloyiaection Physical examination reveals scattered petechia and ecchymoses over his thunk. There is no Jmnpbedenty or hepalosplenomesaly Asoo ar oc hood is negnve His CBC exhibits & erglobin of 13 wf, 120-150 g/L), leukaeye oa of 8 50/mm? (800-13, 500/am) wih ‘anormal differential count, and a platelet count of 10,000/mmt* (150,000—-400,000/nmn’). Answer th patient ha opal thomboostopenieppara 12. "A 38jearoK! womin press with fever, renal sams afratons, and cisuxis. Price! examination reves!s retinal hemorrhages, widespread petechia and.eechymoses, and a positive stoo! {or otal Blood. Tho is no lympbadcnoptiy or fepatoepemrnegaly The CAC report adaes ‘the presence of a normal hemoglobin and leukocyte count. The corrected reticulocyte count is 12%. “The pecpheral ear exhibit merous fagmented RIC, pachromasi, and «reo! umber spite. Aone rare boon conaie vaca chacl conaing psec Grom, Tere ie | 4 poslive sige dipsck for blod and numerous RBCe are present inte sedinet. The sera Mood rea. nitngen i 40 mg (7-18 mpl.) and the sem creatinine is 4 mi. (06-12 sgl). Answer: A~ the patent has tromiboti tromboeytopenic purpura 13, An 18-year-old smoking male presents with deep venous thrombosis (DVT) involving the right lower calf. He has a family history of recurrent DVTs and pulmonary embeli in his motber and material grandfuder, His PLT and PY are normal prior to reesiving n standard dose of heparin fntravenously. The PTT remains normal 1 hour afer infasng heparin. The pathogenesis of his hemostasis abuormality is most likely related to. ‘A. anliphospholipid antibodies B. athrombobemorthagic disoeder . —adeficleney of antithrombin It D. deficiency of vitamin K-dependent factor B. _santibodies directed against heparin Answer: ©: the patient hns hereditary ATH deficiency. Heparin eansot anticosgulate without ATI. The peat isthe lack of increase inthe PT with heparin, 14. The plage of hemoxtiagie sin neon sod wit wartarn therapy i mat csty associated with. ‘A. antibodies directed against warfarin 4B. adrug hypersensitivity reaction C! jeune vasculitis secondary to warfarin D. protein C deficiency in the patient antithrombin If deficiency inthe petient z "Note: This material is copyrighted. All rights reserved. (fdward F. Goljan, M.D.) 2002 ‘Answer: D: heerorygote carriers for protsin © become horonygote when given warfarin in ~6-8 bs ‘wn the halflife of previously y-eaboxylated protein C disappears. Now the paien ie Iypercoaplable, ‘Blood Bank questions 1. A 6S-yearold man in an intensive cat unit ie recovering fom surgery for a ruptured abdominal dol aneurysa. The patent required 12 units of packed ed blood cells prior to surgery in aes to stabilize his blood pressure. On tho Sth postoperative day, he develope fever, sleralicters, and Jw back pain. Physical exam reveals seated rhonchi throughout both lung fel but.no areas of consolidation. & rine sample taken Som his indwelling triary eateter exhibits pyuria. Oftares feolated & col. A CBC reveals 23 gf. drop in his hemoglobin conceataion when compared to his postoperative levels, A direct Commibs' test is positive, No hemoglobin is noted in plasma. The total bilirubin is 4 mg (0.1-L.0 mpi.) with a dtcet (conjugated) biliabin of 04 mp/al. (00-063 mpl. sexu anne aminoicanserase concentration is 20 U/l. (8-20 UI). Which Se ee ee aes eee "File tansfsion reaction 3, Poerefass hepatlen ©. Delayed hemolytic transfusion reaction D, Type I hyporsonstvity reaotion Enjoy ofhypebtinbinnia ‘Extravascular hemolysis ‘smears GD, Feo puta wt layed TR Bo 9 a ertbody ete spt a tip om ‘tte donor RECS. {A blood group G th negative woman wih a negative antibody srean delivers blood group A, Rh positive baby. ‘The buby is tiildly anemic and develops an unconjugated hyperbilirubinemia in the frat 24 hours direct Coombe’ of cord blood returos weakly postive. Spherocytes are noted in the erpere Blood smear. The bby eto! is grossly bloody. An Apt txt is perfarmed onthe stool ample ws reveals adit Remoglobin, The mother’s antibody acreen i nepative. Which of the following best describe the pathogenesis of baby's anemia” SELECT 4 ‘ABO invompataliy Rh inconpatiity “Type IL hypersensitivity exeton, Mother isnot eandidate for Rh immune globulin ABO incompatibility protects aginst th sensitization Positive direct Coombs du to aicD coating babies RBCS ‘Newborn has a Glbleed B,C, B: ABO incompasility does protet against Rh sensitization, bt she sil should reecive Rh immune globulin, Mother's ani A,B 1gG i coating the babies RBCs. The Apt fest distinguishes fetal from adult Hb. The baby has swallowed mommies blood arid does not have a Gtbiced. 3. A group O, Rh negative 32-year-old woman develops fever af 103°F while receiving a unit of O, Rh negative packed zed bio eels aller @lysterviomy. A ransision eaetion workup on the patient reveals the following: A B. i D, E. ER @ Answe ‘Antibody screen negative Direct Coombs’ ‘negative Plasma ‘normal eolor ‘Urinalysis dipstick negative for blood Pretransfusion hemoglobin 7 gL (12.0-16.0 g/dL.) Pasttransfusion hemoglobin 7.5 g/l (12.0-1600 g/dL) Which of the following apply to this case? SELECT 2 4 ‘Note: This material is copyrighted. All rights reserved. (Edward F. Goljan, M.D.) 2002 ‘A. Donor blood contamination with Yersinia enterocolitica B. Type Thypecsenstivity reaction (C. Patient should receive leukoeyte poor blood for further transfusions D. Hemolytic trmsfusion reaction due to antibody destruction of donor RC B, _Pationranti-HL.A antibodies are directed against donor leukooytes Answers: C, FE: febrile reaction: patient must have been expased 1o blood products at some time in her life for anthdies to develop (common in multiparous women who commonly have fetomaternal bleeds daring delivery) 4. An B2-yrold woman with blood group A inadvertently receives blood group B blood. He does not evap a hey tunfsion sion This emo ly et sbsence of isohemagalutinins with old age a defect in celular imemunity Brtons agammagicbuilinemia sntithromn Il deficiency shsent anti-A IgM titers in the donor unit “Answer: A: elderly people lose their isohemaggltinins hence they may not develop hemolytic TRs even ‘witha ABO mismateh, ‘Which of the following is more often associated with Rh hemolytic disease of the newbom due to anti-D antibodies rather than ABO incompatibility? SELECT 2 A. Norislfor hemolytic disease of the newborn during the fst pregency 1B. Positive direet Coombs onthe babies eord RBCS .__Spheroeytes in te newborn's periperal blood D. Severe anemia requiring blood transfusion ‘Unconjugated hyperbiliabinemia ¥._ Type hypersensitivity reaction “answers: A, Di mother’s get sensitized against Rh antigen i Ui first Rh incompatible pregnancy, but ‘this doesnot affect that baby but future babies 5. A blood. group O, Rh negative 65 oléayearold man with known diverticulosis presents with & amassive lower GI bleed (hematochezia). He has an estimated blood los of greater than 600 ru. cover the last few hours. An intravenous line with 0.9% normal sine is in place, while blood is ‘bing crossmatsed in he blood bank, His High is 6 p/. Physial exam reveals cold, clamsay skin, ‘blood pressure of 70/40 mm Hg, and a weak pase with a sate of 120 bpm, Owing toa shorlage of ©, Rh negative blood only 1 ofthe S nits of packed RBCs are O, Rh negative, while the remaining 4 wis are O, Rh positive. The patient has no history ofa previous tansfsion or exposure to blood products. Midway’ throug the second nit of blood, which i Uke first ofthe four nits of O, Rh positive blood, he develops wheezing, dyepnea, and swelling of his face. ‘The transfusion is ‘topped, a subcutaneous injection of aqueous epinephrine at a 1:1000 dilution is given to the ‘tient, and a transfusion reation work-up is ordered, The transfusion resction workup on poste transfsion specimen of patient blood is as follows: FEAR ‘Patient ferperatur; 1000°R Patient blood pressure: 60/40 ran He Patient pulse: 130 bpm Patient plasma: clear ‘Patient antibody sereen: negative Patient direct Coombs’ negative atient rine: negative dipstick for blood. ‘Thepaogsns of his ptt tants i stl led. SELECT 2 ant antibodies destroying donor D antigen positive RBCs 3 jaca REA notes macy pina ocean © A ptientFa-medited reaction against a donor allergen D__alype hypersensitivity reaction 2s ‘Note: ‘This material is copysighted. ll rights reserved, (Edward F, Goljen, M.D.) 2002 E _anintrvaseular hemolytic anemia Answers: C,D: allesic reaction 7. A blood. group 0, Rh negative worm. with 9 negative antibody soxeen and no previous ‘sdministration of Rh immune globulin ducing hec pregnancy delivers a blood group B, Rh positive aby. ‘The baby develops unconjugated yperbiliubincmia a few hows after birth. The otogeeni af the ayo mo ley rate ‘maternal anti-D antibodies destroying the babies Rh positive RBC intravascular hemolysis of fetal RBCS by sn JeM antibodies intravascular hemolysis of fetal RBCs by anti-A. IgG antibodies cexiravasoular hemolysis of fetal RBCs by ant IgG antibodies extravascular hemolysis of feta RBCs by anti-A, TaG antibodies Answer: E: the mother is ABO and Rh incompatible with het baby. She bas not been previously sensitized to D antigen, soit cannot explain the baby's jaundice, therefore, ABO hemolytic disease ‘ofthe newinom is the most likey ease, 8 Which of the following characterizes « major crossmatch? SELKCT'3 A. Prevents post-ransusion hepatitis due to hepatitis C B. Doce not gusrante survival af the infused donor RECs ! Rules out the possibility ofa febrile transfusion reaction 1D, Does not prevent patent antibodies developing against donor RBC antigens 1B. Prevents antibodies in the donor from destroying patient RBCs F. _Detests the presence of patent antibodies against donor RAC antigens Answers: B, D, F-amajor crossmatch involves mixing patient serum with donor RBS to se if they re comstile toms 9-12 ‘A. Blood group A BL Blood group AB © Blood group B D. _ Blood group 0 9. A AS-yeareld man with exertional dyepnes and extreme fatigue ad u lng history of duodenal “cer diseases noted to have dare back, try stools. Physical exatm revels pale conjunctiva and a ‘wide pulse pressure. Laboratory studies reveal a6 ll hemoglobin and alow MCY. Danor blood {or transfusion of this patent based on the type of peptic wleerdiscase he has would most likely Ihave which ofthe shove forward and back type esuls inthe blood bank? “Answer: D: blood group © people ean only receive blood group O. Remember that the forward type with antisera identities the blood group, while the back (ype using tet RECs, identifies the anibody ‘ssonated with the blood group. 10. A.32-yearold woman and her husband tuve 10 eildren. Her husband's blood group is A. All of their children are ether A, B, or AB. In order to explain the blood groups in her ehiléren, you ‘would expect the woman to have which of the ebove forward.and back fype results on x sample of Yer blood? Answer: B: she must be AB to have A, B, or AB children. AB people are universal recipients, since they Tack antibodies. 11, A 49:yearold has a gastric adenocarcinoma. Based on his diagnosis, which of the above blood! soup types is best for tanslsing the patient? “Answer: A: gastric adenocarcinomas are usualy seen in group A patients It is always best to give the patient the same ABO group rather than O packed RBCs, 12, A patient with this blood group could develop a hemolytic transfusion reaction after receiving AB. ‘blood or A blood ‘Answer: C: the patient must be blood group B. RODE > 6 ‘Note: This material is copyrighted. Al rights reserved. (Rdward F. Goljen, M.D) 2002 13. A phlebotomist inadvertently sticks himself with a needle after drawing blood froma patient with AIDS. Which of the following infections is the phlebotomist at most risk for contracting? A HV B. Hepatitis A ©. Hepatitis D. Hepatitis C E, Syphilis Answer: A: 1300 chance of becoming HIV positive 14. "A phlobolonit inadvertently sivks uml? witha needle afer diawing blood Bom a pct. Which af he allowing infetons i the phcbotamis at most far eoneating? A HV B, Depatis A Answer: C: HBV has the greatest viril loed én blood of all viruses. You canot get syphilis ftom transfsed blood. 15. Which of the following is the most common antibody encountered in clinical practice? A. AntPHAV-IgG B. Amt-HIBs C AntiHHCV-IgG D. Ani-CMV E. _ Heterophile antibodies Answer: D: iti almost impossible to find blood that is negative for antibodies against CMV. This was ‘ett withthe National Red Cross 16 A puient sho as chronic hepatitis and hasbeen transfused in the past most likely has wnibodiee directed agaist... A. bepatitis A BL hepatitis hepatitis © Dd. HIV E cMy Answer: C: HCV is the MCC of transfusion hepatitis Cardiovascular questions 1. A Te-year-old man presents with & sudden onset of left lank pain, Tn dhe emergency room, the patient is hypotensive. A pulsatile mass is palpated in the abdomen. Which of the following is ‘MOST reaponsible forthe pathogenesis ofthis patient's condition? A. Atherosclerosis BL Adefect in bein CA defeat in collagen 1D. Long-standing hypertension E, Immune destruction of elastic tissue ‘Answer: A- ruptured abdominal aortic aneurysm, Note the rupture triad of left flank pai ‘end pulsatile mass, a "Note: This material is copyrighted, All rights reserved. (award F. Goljan, M.D.) 2002 2 A-42:yearold man with a history of eardiae death in his family prescats with a sudden onset of severe, stostemal chest pain with radiation of the pain into hit bask. is Ieft pulse is abvent. A high pitched diastole blowing murmur that ineasos with expiration is hoard immediately afer Sy. “There is widening ofthe aortic knob on a chest x-ray, The mechanism for his patient's condition i MOST CLOSELY related to A. atherosclerosis Ba defect in fibitin ©. Adefet in collagen D, aninfeotious process . — snacute myocardial infiretion Answer: B dissecting aortic aneurysm. Note the ype of pain radiation into the back, absent pulse, ‘murmur of aocti rogugitation and widening ofthe aor knob, 3. A 26-year-old woman presents with a history of chest palpitstions particularly when anxious. Physical exam reveals a midsystolic ejection eick followed by a mum. The chek aud murmur ‘move closer to Si when the patient is standing and. closer to S: whon the lying down. Tho mechanism for this patent's valvular disorder ¢ MOST CLOSELY roated to. defect in fibrin ‘an infectious process ‘immunologic damage riysomatois degeneration 2 defect in collagen syuhesis “Answer: D~ mitral valve profanee. When the LVEDV is doorensed (standing, anxiety) the click and murmur come closer fo SI. When the L:VEDV is increased (lying down, clenching fs), the click smd murmur come eloser to S2. 4. A2B-year-old patient has a family history of ucden cardiac death ata young age, The patients a syatalic jection murmur tat decreases in intensity when the patient is Iying down ad increases ia inteasity whea standing up. An echocardiogram reveals alnotmal movement of the anterior ital valve leflet agsinst an asymmotrially thickened interventricular septum. The patient MOST LIKELY bas A. ital valve prolapse B. hypertrophic cardiomyopathy ©. congenital biouspid sowie valve 1D. acardise myxoma ofthe eft ateium E. _ infective endocarditis involving the mil valve Answer: B- hypertrophic cardiomyopathy- MCC of sudden death in young people, The asymmetric septum has abnormal conduction bundles. 5. 65-year-old man on he Sth day of hospitalization for an asta steror myooerdial infarction has "carags of chest pin nan nse Boh CRM and woponit md. The patent MOST POOR A. papillary muscle dysfunction BL aright ventricular infarct © aventricular aneurysm 1D. amyocardial rupture _reinfaretion ‘Answer: E~rcappearance of CK-MB after 3 dais the definition of reinfartion 6 Whi of he following is prent in BOTH ef ed ght se ea uae? ‘S; heart sound Plow ebopise ©. Pulmonary edema D. Neck vein distention E. Dependent pitting edema ‘Note: This atrial is copyrighted. Al rights reserved. (Edward F, Golf, M.D) 2002 Answer: A~ choice B in LHF, choice © in LHF, choice Din. RHF, choiee Ein RHF) 7. You would expect « tent with an arial septal defect 1 have which of the following oxygen saluration (8:0) findings obtained by cardio catheterization? YNoratal Patient Patient Retient Patient Patient 8a0,% A x fr Ss Rightatsiom 75 5 no - 8 4. Rightventtcle 75, 80 mo % 5 8 Pulmonary art 75, 80 mo 8 80 Pulmonary vein 95 95 9 95 SS Left ventricle 99 95 9 95 80 ‘Aodta 95. 95 9 «95D Lgapeeceenarieo ‘Answer: B~ isan ASD; A= VSD, B= ASD, C=1DA, D~‘Tetrloyy = transpsifon. Note that 4 sep up in oxygen saturation in a chamber or veeel usually means & left to right shunt, while sep down in orygen stration indicates aright to let shunt (cymotic CHD) 8, A S0syearold man witha viral myocarditis who develops hypotension, neck vein distention, a drop in blood pressure on inspiration, and moffed heart sounds most kely has... a dissecting sore nneury Answer: D~ Beak’ iad of pericardial effin, Fst stop isto do an echocardiogram Ipositve, then a ‘pescardocentesis must he dono. tems S01 A, Anvtiostenosis 3 ‘Aortiergurgitation ©. Mital stenosis D. Mita regurgitation B._Tiouspid regurgitation 9. ATO-yenr-old man vith diminished pulses and history of angina and syncope with exercise has ‘ection type murmur radiating into the cat arteien ‘Answer: A- aor stenosis is the MC valvular lesion asnciated with angina and syncope with exercise. in lsa the MCC of mioangiopstic hemolytic anes 10. A S8ear-old man with leftsided heart failure has an $3 and $4 heart sound sd « pansystolic murmur located tthe apex that increases with expiration Answer: 1D~ mitral regurgitation in heart flue is duct steiching of he msital valvering due fo volume ‘overoad inthe left venticle 11, -A20-yearold intravenous drug dbuser has foer,« giant e-v wave an a pansystolie mem slong, the let stemal border that increases with inspiration Answer: F-iiuspid regargtation isthe MC valvular Iesion 1VDA. tis det 8. aureus. Remember that allright sided murmurs and abnormal heart sounds increase on inspiration, while the reverses tre for ef sided mrmars and abnormal heart sours 12, Which of the following pes of hepatitis is associated wit a vasculitis du wo classical polyatrts » “Note: This material i copyrighted. All rights reserved. (dward F. Goljan, M.D.) 2002 ‘Answer: B= polyarerits nodosa. High association with HBsAg. It is mot associated with p-ANCA ‘(microscopic polyangits is). ANCA is scen in Weaener's granulomatosis. 13, A year-old boy presents with a low-grade fever, atkralgis, colicky abdominal pain, and palpable purpric rash limited tothe lower extremities. Laboratory studies teva « guaiac-pasitive ‘oo, urinalysis with red blood cell (EC) casts, ear, andl protein anda CBC with ‘normal Th, Hot, and platelet count. Which of the following is the most likely diagnosis? ‘AL Idiopathic thrombocytopenia purpura (TP) 2B. _ Systemic lupus erythematosus (SLE) C.__Poststrepiococeal glomerulonepbritis D, Rocky Mountain spotied fever E. _Hlenosh-Schénloin vasculitis ‘Amswor: E- it is a type IL inimunocomplex disease. Mott clinicians think that RSP and YeA slomerulonephets re the same, Respiratory questions 1. asshich ofc fllowing cnc sccraios involving pation with hung disease would you expect faction studies to exhibit decreased compliance ineeased elasticity, an an increased Fav ua/FVCauto? A. Garcaroli child with reeurentreapratory infections and storie B. 28yearold non-smoking wale with bilateral lower lobe emphysema G. _S6sjoaceld smoker with productive cous dyspaea, wed eyanoss D. iOsyearold grt wth bronchial sthne-rgtring simi steroids 1B. _S93parald dyspnec Afean American wih blteral iar nodes Answer: =the PPTs deseribe a esrctve lang étonier, in his oa, sarcoidosis. Choice A describes fic fibrosis, B deseribes cd-antitypsin deficiency, C dessribes COPD, and D dexebes ‘rosea! ath 2. _AdSuje od wontn 24 hours post choleystectny develops fever and Aspnes. Physioal exam Tevesitdearasedpercuision, increased tactile fremsts, nnd eereaged breath sounds in the rah lower lobe. The daphzagm is clevated and thr i inspiratory Ig on he ih sie, The patent MOST LIKELY hs. ‘A. atelectasis B. ahigabseess S D. a puimoney infection Bs mpontncous paeumathoes Answer: A- these are the classic physical findings of atelectasis. They are similar to consolidations noted ‘in bronchapresmonia and lobar pacumonia. Remember at fover inthe first 248 hrs after surgery is aelectass ‘An afebrile 23-year-old man develops a sudden onset of left-sided, stabbing chest pain with s4yomen. Physical exam of the left chest reveals hyperesonance to percussion, deviation of the teaches fo the left, clevation ofthe diaphragm, descend ‘cil Semis, and devresed breath sounds. The MOST LIKELY diagnosis. ‘A. pleural ffasion 1B, Sronchopnotmonia tension pneumothorax D. _apulmonay infarction .spontneous poeumathorax Answer: E~in spntaneos petmothora, the hing collapses causing the presse in the pleural cavity to eth same as hatin he atmospliec A tension peuinothore fear in the pear, which opens and admits into the pleial cavity during inaprtion and close ding expiration. This nteass 20 (Note: This material is copyrighted. All rights reserved. (Edward F. Goljan, M:D.) 2002 the postive pressure in the pleural eavity and shifts the mediastinum and trachea othe contralateral side Also 0eours when ascending tothe surfiee in scuba diving. ‘Aznewbom child develops dyapnce, tachypnea, intercostal mascl retraction, and eyanosis hours afer bith. The mother developed gestational diabetes melts and was it poor glyecnie contol ‘throughout the pregnancy. A chest x-ray reveals a “ground glass” appearance in both lungs. The -cimary mechanism for this patent's respiratory problem i. ‘A. sppiration of anmiotic fuid B. group B streptococcus pneumonia (C. decreased production of surfuotant D. Chlamydia tackamatis pneumonia E. _ hear falute from congenital bear discise “Answer: C-~ this ig dlassic RDS. Poor glycemio control in the mother leads to hyperglycemia in the fetus 5. and release of insulin, which inhibits surfactant synthesis by type I pmeunnooytes. Tnoreased tate freritus Neutrophilie leukocytosis ‘Answer: B- this question compares atypical with typiea! pneumonia, the former most often due to M. ‘Pheumonise and the fter'S. pneumoniae. The former isan interstitial pneumonia and the latter a ‘consolidation type of prcumonia, All other choices are those of typical pneunonia and would not ‘be expeoted in an atypical pneumonia {A 58-year-old smoker presents with Weight loss and cough. Physical exam revels a mild lid lag on the left and 2 pinpoint pupil, scattered sibilant shonchi throuphout all ung fields tua clear with covghtg, an an inteeed anteoponeor dame. Based on Use finns, you pect the patient has... A. aPancoast tumor B. a thorsci outlet syndrome © the superioe vena caval syndrome D, obstructive lang disease without primary cancer E. _ obstructive ang disease with metastatic eancer from another primary site Answer: A~ a supztior suleus tumor, usually squamous eancer, invades the lower part of th brachial 7 ‘plexus and destroys the superior cervical ganglion leading to a classic Horner's syndrome. ‘A S-year-old man with urinary retention secondary to prostatic hyperplasia, develops spiking ‘ever, and tachypnes. Physical exam reveals intercostal muscle retrations and bilateral inepratory cracides. A chest x-ray exhibits bilateral interstitial and alveolar inflates. ABGs demonstrate severe hypoxemia. You expect the blood culture reveals... _gram pasitive diplovocci ‘gram negative cplocoec: gram positive coosi ‘gram negative rods sam positive rods Ppp ‘Answer: D~ the patient has gram negative sepsis due to E.coli (gram negative rod) which progressed into ARDS. Indwelling urinary catheters are the MCC of this type of sepsis, ARDS and DIC are ‘complications of gram negative sepsis. Inspiratory stridor is commonly associated with A. arespiratory syncytial vies infection BL a parainfluenca virus infection aspirin induced asthma at Note: This material is copyrighted. All rights reserved. (Faward F_Goljan, MD.) 2002 D. shinovina infections E. — ehoanal atresia Answer: B- croup or laryngotraheobronchitis is due fo parunfluenza virus, J-eads to obstoction inthe “trachea, which on xray appear ikea "stzeple” ofa church Acute epilatis dus to 2. inluenaae aso produces inspiratory stidor. 9. Cllamyaia trachomatis and the eopiraory syncytial virus are BOTH commonly associated with, ‘A anintemtital ype of paeumonia BL _laryngotacheobronchits(reup) C. _theresprstory dstess syndrome D. typical community-aoquired pneumonia E. _hospital-acquired (nosocomial) paeumonia, Answer: A~ RSV is the MCC of pneumonia snd bronchiolitis in ehilden, . rackomats produces @ staccato cough, wheezing with tmpping of sr, and eosinophilia. There is no fever an blaeral coujunetvitis is usually preset s well 10. Which of the following is more often asvosiated with Klebsiella pneumoniae than Pueudomonas aeruginosa’? A, Upper lobe cavitation B, Green-coloved sputum ©. Association with eyed Sbrosie D. Association with respirators E. _ Productive cough Answer: A~ choices B, C, D are festures of P. aeruginosa. Both have productive caugh (choice’B). Tl. a.a 30-year-old inan wo lives in Tennessee, you would expect a ealcifiod solitary coin lesion in Saker a forcign body sp old granuloms metastatic cancer ‘a primary lung cancer ‘bronchial hamartoma ‘B most solitary nodules in the lungs ate granulomas dnd in the Ohio-Temnessee valley, histoplasmosis is the most likely cause. Tn the Southwest it would be enceldioidemyeosis, in the ‘Southeast, bastomycosis, and in the Northeast, cryptococcosis. 12, A 55-year-old non-smoking cual worker has arthritis and nodular lesions inthe lungs, His PPD skin (est is negative. You suspect the patient has.. A. systemic lapus erythematans BL Coplan's syndrome CL —-melastitic lang disease De primary lang eaoer 1B. miliary tuberentosis ‘Answer: B- this underscores the association of rheumatoid nodules in the kines with » pneumoconiosis, ‘n this ease coal worker's pneurnoconiosis, 13, Ina 62-year-old man who has been a roofer for 25 years and a smoker for 10 years, which of the {lowing careers would he be ms ily prone io developing? § DOE > Auswi Pancreatic cancer ‘Answer: B- the patient has had asbestos exposure, since soofing material at that time had asbestos in its composition. Whether the patient was a smoker or nol, a primary lung canoer would be more ‘common than a mesothelioma. Mesotholiomas take much longer to develop than lung eancer and 32 3 i ‘Noto: This material is copyrighted. All rights reserved. (Bdward F. Gofjan, MID.) 2002 dave no smoking relationship. The MIC: Tung problem asociated with asbestos i a honign pleural plague, 14, Abrige painter in Brooke, New York develops a pulmonary inftate, Which of the fllowing ‘groups of pathogens aceon your differential Hist? ‘A. Histoplanma capsulatum and Cryptococcus neoformans 1B, Grypiocooeus neofoninans aid Coccidiides itis . _Blastomyecs dermatitis and Histoplasma cepsuatum D. Aspergillosfumigats and Cocciioidesinmits B, Poeumoeysts cain and Aspergillus fumigatus ‘Answer A. Histoplasma expaulsum and Cryptococcus neofomuns, Starlings carry Histoplasma and ‘pigeons carry Cryptococcus and bth oost on bridges. 15, Which of the following i « hypersensitivity pucunsnitrthat pearly ocours fn textile workers? Silo filler’ disease Bagassosis Farmer's lang Byssinosis Bi _ Sareoidosis “Answer: D- exposure 1 }inen, hemp, er ction produc my cause a ypersenstivity reaction inthe tng ‘vith dyspnea snd inflates Patents improve over the weekend and get depressed on Monday “Kaowia tat they wil be sc, ene the term "Monday morning bles" 16, Which of the following is a kyperseasitnaty pneumonitis that commonly occurs in farmers who center closed oom with fermenting com? ‘A. Silo filler’ disease B. Bagassosis €. Farmer's lung D. Byssinesis B. Sacoidosis ‘Answer: A. Silo file's disease i related to nitrogen dioxide is given offby the fermenting cor. Iteauses ‘an immediate reaction in the Iungs. Farmer's lung is due to a hypersensitivity reaction against thermophilic ntinomycets aut in the filds. Bagassosis is reaotion against sugar cane. 17, Tea patient is breathing room air (21% oxygen), the arterial PCO. is $0 mm Hg, and the arterial ‘PaO, i840 mam Hg, what is the Alveolar-arierial (A-a) gradient of te patient? A 10. BIS e 2 D2 B30 ‘Amswer A~ the formula to caloulate the PAO; is PAO: = % O; (713) - PaCO¥/0.8. In this problem the PAO, = 0.21 (713) - 80/08 = 50, therefore, Ara = 50 - 40 = 10, A medically significant A-x ‘gradient is > 30 mm Hg. Any Inng disease would be expected to increase the gradient e.., ‘Pneumonia, COPD, saeoidosis, ARDS. Hypoxemia due to conditions not directly lavolving the ‘ngs have a normal gradient For example, 33 ‘Note: This material is copyrighted. All rights reserved. (Edward F. Golan, M.D.) 2002 ‘Gastrointestinal questions Items 1-6 ROmpEDOm> g = 1, An afebrile 22-year-old man and several other members of his family developed severe vomiting, ‘vithout diarrhea ~1~6 hours after eating potato salad at a pieni, They all recovered uneventflly 12-24 honrs later. Answer: G-this is ve to. aureus producing’ toxin which i ingested inthe food, Culture the food, not the stool 2, A 23-yeacold man developed explosive, watery daeres with blood, leukoeytes, snd mueus ~3 days after cating chicken that was improperly cooked. Comma to S-shaped organisms are noted in the fecal sear of stool along with RBCs and Teskoeyts. Answer: E- Campilobacier isthe MCC of invasive gastoenirts th the US. IE ean produse crypt sbecesaoe and simaate UC. 3. A febrile 10sear-old child presents with severe right lower quadeat pain that is intrreted by the attending physician as acue appendicitis. At laparotomy, the surgeon notes tat the append is oral. However, the mesenteric Iymph nodes are markedly enlarged and have focal areas of ricronbseess formation on eut section, ‘Answer: F- Yersinia is'a cause of mesenteric adeniti, granulomatous disorder, tat-san be confused ‘with acute appendicitis. 4, A.29-yearold man. develops watery diarthea and volume depletion shortly after visiting the Gulf Coast states Answer: A~ Vibrio cholerae produces a secretory diarrhea de to toxin stimulation of e-AMP. 'S. Ona tip outside ofthe country, a man develops a high fever associated with bradyeardia absolute ncuropenia, and splenomegaly. A blood culture is positive fora gram nogative orgasm. Answer: D- Salmonella yphi procuces this clasic triad during the second week of infection, A bursa, ‘A23-yr-old man develops vomiting and diarrhea after cating tefted rice and tacos. Gram positive rods are noted in the stool, ‘Answer: B~B, cereus produces a preformed toxin, Salmonella enteritidis isthe MCC of food poisoning Items 7-8 A. Yersinia enterocolitis B. Staphylococcus aureus ©. Rierotoxigente col D. Ghyptosporiium parvum E. Mycobacterium avium indracllulare 7. A 2Byearold man with AIDS presents with chronic, reourent, profnse, nonbloody, watery diarehes. An Entero“Test (string et) reveals oocysts tat are partially acid-fast postive, Answer: D- Ciyptosparidium, a sporozon, i the MC pathogen in AIDS diarrhea B.A 25.yearold medical student during Spring break outside the county develope fever, vositing, adominal ramps, and watery darhea ~14 hours after eating afew tacos purchased from a street, vendor. He recovers uneventflly in 4872 hours. Amswer: C~it isa sceretory diarrhea dust heat stable toxin ha stimulate guanylate cyclase M ‘Note: This material is copyrighted. AI rights reserved. (Baward F. Goljan, M.D} 2002 9. Céynophagia in a HIV-positive 28-year-old mau with white plaque: ‘buccal mucosa MOST LIKELY has an AIDS-defining lesion eaused by. A. Epetein-Bare virus B. Candida aibicans ‘material on his tongue and ‘hash 10, An afebrile 25-yearold medical student presents with itermitent complaints of eft and right ower quadrant abdominal pain and distention associaled with allemating periods of mucoid the. = eee .exophngws Do Somech rec newer: Belt 13. Which ofthe following characterizes wleerative colitis rather thas Crain's disease? iseontinuous spread ‘Toxic megacolon Fistula formation Periaual disease ‘Obstrution Answer: Ball the other choices characterize CD 14, A 38-year-old Asian woman hes a long history of explosive diarchea and abdominal distention after ‘ating dairy products. The pathogenesis ofthis patient's diarrhea is MOST CLOSELY related to. ‘A. antigladin antibodies B. activation of eyclic AMP ‘C. iniraluminal osmotically active solutes 1D, mucosal injury with increased permeability E. loss of the absorptive surface ofthe small howel ‘Answer: C- lactase deficiency. Laciase is a disaecharidase and a brush border enzyme It produces an ‘osmotic type of diarrhea. 35 ‘Note: Thi misters i copyrighted. All rights reserved. (ldward F. Goljan, McD) 2002 15. 4 G2-year old man smoker presents with weight loss, « dragging sensation in bie right upper ‘quadant, and crampy lef lower quadrant abdominal pain. He has alternating bouts of constipation and diarrhea. In addition, he states that blood coats and is mixed in with his stools. He somotimes ‘has pin with defecation. There is mild hepatomegaly. The reetal exam reveals nion-thrombosed ‘external hemorthoids. The stool is guatac postive. 4 complete blood cell count (CBC) reveals 2 ‘mild microeytic artemis. The MOST LIKELY eause for this patient's condition is... sngiodysplasia ‘an anal fissore diverticulitis ‘hemorrhoids ‘colon cancer ‘there i also evidence of liver metastasis .geceld me pees with bing of he as, mtery dais, weight ow, nd ple ‘in is liver, The primary sive forthe cancer tt led tothe above symptom complex is tong liver ‘stomact, Peneege appendix ‘terminal ileum. ‘Answer: B- the patient Its the éafvinoid syndrome. Appendiceal carcinoids do not metastasize to the liver, 0 it would not produce the syndrome. 17, Hematochezia in elderly patients that cannot be identified with « barium enema study is most ‘commonly due toa disorder located inthe. AL rectum) BL stomach, €. sigmoid colon D. proximal duodenum 1. _ Gevimand ascending colon Answer: B- the patent lus angiodyplasa, There is a relationship with aortic stenosis and von Willebrand's disease. Diverticulosis isthe MCC of hematochesia, however, the barium enema is soem 18, A febrile 58-year-old man witha Jong history ofchroni constipation complains af «steady pain in ‘left lowers Heb fad Segue aca of pn nese ae for cevel ments end ‘one episode of bloody stools that spantancously resolved. Flysical exam reveals rebound tenderaess end a palpable mass in the left lower quadrant. A stool guaiao fs negative. A CBC éxinbits am absolute neutrophilic leukoeytois and let shift. A flat plate i reported to show vo air under the disphragm. The most likely diagnass is AL volvals B. colon cancer CC. ischemie colitis D. acute diverticulitis E, _ small bowel infarction Answer: D~ acute diverticulitis presents as a "left-sided eppendicits" 19. A 584yrold woman complains of epigastic pain, weight loss, vomiting of eoffee ground-like ‘steal, and dark black sticky stole. Physical exam reveals cpigasiic pain to deep palpation and non-tender, hard left supraclavicular mph nodes. Both ovaries are enlarged and firm on bimanual pelvic exam. Which of the following soearios best explains the signs and symptoms in this patient? ‘A. Stomach cancer with metastasis tothe ovaries B. Primary ovarian cancer with metastasis 1o the stomach 36 "Nate: This material is copyrighted. All rights reserved. (Edward F. Goljan, M:D,) 2002 © Primary pancreatic cancer with metastasis tothe ovaries 1D. Malignant lymphoma ofthe stomach with metastasis to the ovaries E. Primary ovarian eancer with metastasis to the supraclavicular lymph nodes Answer: A~ note Virchow’s node and the history of weight loss. Remember that « postmenopausal ‘woman should not have palpable ovaries. It could by primary ovarien cancer, or as in this eese, ‘metastasis fo the ovaries (Krukenberg tumor) fom stomach cancer. 20. A 65-year-old man with a chronic archythmin with an imegularly ireyular beat presents with a ‘sudden onset of severe abdominal pain associated with vomiting, abdominal distention, and bloody 50% ofthe total it A. Gilbert's syndrome B) Cheeni vial hepatitis ©. CaiplerNajar mdcome D. Stone inthe orton bile duct E. _Extavasnlar hemolytic anemia Answer: D- CB >50% shvays indiates abststive jaundice. Choice A (Gilberts) would have a Cis “20% (dereased pak and conjogaion), choice B would have » CR 20-50% (publom with uptake/conjugation and necrosis of bile ducts causing release of CB), choice C (Crigler-Najjar) ‘would have a CB <20% (deficiency of conjugation enzymes), choice B (EHA) wosld have a CB <20%, due to macrophage destruction of RBCs and UCB ss the end-product of Hgb degradation. 1. An aeble 62yearald an wth 4 history of alcohol, stoking, ane chronic paneresitis resents with weight los, a slow onoet of painless jaundice, and 2 normocytic ane, Paysial Eramrewalsa palpable palbiadder anda lipi-colereé stool. THe patient MOST LIKELY has ‘A. epatocellla eareinoma carcino of the eallbladder 38 ‘Note: This material is eupyrighted, All rights reserved. (Pdward F. Goljan, M.D.) 2002 ©. primary sclerosing colanits D, fslonetin th conn bie dct Carcinoma ofthe head of panoeas Ansner: noe the otrctive signs of ght oloed stool and palble GB (Courvosier' sign) 8. A.38yeacold woman preents wit orono liver sass, rtnishrbrown deposi inthe inbus of degenerative seas inthe lenticular nuclei ‘Answers: , the patient has Wilson's disease, Note that total coppers low beemuse ceruloplasmin, dhe ‘ining protein for copper is low. The Kayser-Fleschec ving is deserfbed. 9, A-Ss-year-old woman presets with generalized pruritus, nn-ender hopatomcgaly, and yellow ‘papalar lesions seattered over her skin. She has marked elevation of serum alkaline phosphatase and yglutamyltranferase, 2 normal total bitin, slightly clovated serum transaminases, and a severe hyperlipidemia. An endoscopic retrograde cholangiopancrestography (ERCP) stuly of the ‘common bile duc is negative for stones. Which of the following nee -expected Dinings i this ‘patient? SELECT3 A. Hyperriplyeeridemia 1, Biliary cirrhosis C.—Antimitoobondrial antibodies D. _Tnereased serum IgG levels F, Granulomatous inflammation Answers: B, C; E~ the patient hus peimary biliary circhosis, where there is granulomatous destruction of bile dicts im the porta triads. IgM Tevels are inoreasedas well as cholesterol dhe to obstruction to hile, 10. “Which of the folowing complications is a direet by-product of portal hypertension secondary 0 sleobolic einhosis? SELECT 3 AD Ascites B. Gynecomastia Esophageal varices D. Hepatic encephalopathy . _Petiumbilical caput medusae Answers: A, C, E- gynecomastia is due to estrogen exccas and encephalopathy duc to an inerase in atimnia nd false neuotensmiters 11. A.2S.yearold wom develops jamie, The conjugated bilirubin action is > SOY of the Sots hitintin and the serum alkaline phosphateee is markedly elevated. You eupeet the patient i aking, ‘wich of the following medications? SELECT 1 A. Tsoniazid B. Allopurinot © Tetracycline D. Acetaminophen F. _Oral contraceptives ‘Answer: E—birth control pills produce intrahepatic cholestasis 12, An afebrile high school wrestler develops jaundice, ight colored stools, dark urine, and mild non ‘ender hepatomegaly. The urine is negative for urobilinogen and positive for bilirubin. The serum alkaline phosphatase is markedly elevated. His hepatitis serology tests and bcterophile antibody test are both negative. You suspect the patient has. SELECT 1 ‘A. astone in the eommon bile duct B. been taking anabolic steroids 39 ‘Note: This material is copyrighted. All rights reserved. (Edward F, Goljan, M:D) 2002 C. infectious mononucleosis D, non A, non-B hepatitis E, clonic cholecyatiis Answer: B- similar to question 11, anabolic steroids also produce intrahepatic cholestasis 13, A A2yearold Asian man with postncerotic necrosis secondary to chronic hepatitis B has a low ‘grade fever, woight loss, and a rapid development of ascites. A peritoneal tap reveals bloody ascitic: uid, The finid WBC count is normal end bacteria are not present. on Gram stain, The ‘afetoprotein level is 500 ng/ml (normal < 6 ng/ml). The MOST LIKELY diaguosis is SELECT 1 ‘A. prirmuy gallbladder eancer 'B.hepatocellular carcinoma C._mmetastate liver disease Answer: thsi the classi history for HCC. Note the nore i AIP. 14. A patient Iiving in 2 Basque community in Southern Arizona presents with a history of recurrent sight upper quadrant pin. He isa sheepherder and basa dog that helps him keep tho sheep togetbes. “An ultrasound of his fiver reveals a eyatic mass inthe liver with ealcifications inthe lining of the eyst Which ofthe following additional findings would you expect in ths patient? SBLECT 1 ‘AD His dogs an intermediate host BL Hrejsa definitive host C._Heate an infected sheep D. _ Hisdog ate an infected sheep ‘Answer: D- the patient has echinococoosis, ‘The eye for the parasite is gg to larva to st, The host ‘with the larva is always the intermediate host (infeated sheep, sheepherder), while the host with the ‘alts is the definitive host (dog who ale the infected deep. The sheepherder got the disease from ‘the dog who has the aduls that produce eggs. The eggs develop into the lara ithe shepherd, ‘who isthe intermediate host. Rasque's and Greek’ ae famous sheepherders. 15, A weighilifter develops a sudden onset of sixiominal pain along with hypovolemfe shoek At surgery, his abdominal cavity is filled up with blood. The cauce of the intraabdominal bleed is most Tikely associated with which of the following? SELECT 2 A Anabolio steroids B. Hepatocellular carcinoma ©. _Livorcell adenoma D. Cavernous hemangioma in the liver BE. Staining t stool Answers: A, C~ alvays assume that weightiftes, wrestlers, or professional football players are on steroids. Anabolic steroids and estrogen not only produce intabepatc cholestasis. but also produce ‘benign tumors called iver cell adenomas. They hive as tendeney to mipfire and produce {intraperitoneal hemorthage. 16, A3S-year-old executive ofa up and coming computer company develops a sudden anset of severe cpigussic pain with radiation of the pain into the back. He deseribes the pain as Knifelike, Is audition, he feels nauseous and has vomited non-bile stained fluid on two oseasions. Physical exam reveals tendemess in the epigastric area but 10 rebound tendemess. There is mild tender hepatomegaly. A fat plate of the abdomen reveals x dilated loop of small bowel inthe left upper quadrant An tltasound reveals no stones inthe gallbladder and a normal diameter ofthe coramon File duct. A siool gusiac is negative. Serum AST is higher than serum ALT. What test is most indicated in this patient? SELECT 1 ‘A. Upper gastrointestinal endoscopy B. Serum amylase andr lipase CAIDA (radionuclide) scan to R/O a oystio duct stone 40 Note This aera is copyrighted. All ight reserve. (award F., Goljan, MD.) 2002 D. Hepatitis serologies E, Surgical consult to R/O small bowel obstruction ‘Answer: B- the patient is an aleoholic (AST> AILT) with aoute panereatitis (pain radiating into back and sentinel loop due to localized ileus), onal questions L ‘An 82-year-old man presents with lower back pein and édmplaints of problems with voiding urn. ‘There is point tendemess over the lower lumbar vertebra, His bladder is percussed at the level of Auswer: ©- the patient has metastatic prostate eaucer, ‘The best first test is a reetsl, which vould 2 eftnitely identify cancer. The her tests can then be ordered. ‘A 28-year oldman with a history of removal of «rightryplorchid testis asa child is noted to have 8 painless enlargement ofthe remaining testicle inthe left seotal sac, The testicle is enlarged and does i ranilluinate. The patie most likely has a... A. hydocele 1B. seminoms C. varicocele D. yolksac tumor FE. _cheriocareinoma Ausswer: B~ the key isthe eryptorchidism. Even the uninvolved testicle is at isk. 3 “A 66-year-old man with a 45 pack year history of smoking presents with hematuria, fever, and a ‘llpable mass in the Jeft lower quadrant. A chest xay reveals multiple nodular masses in the Jungs. These findings most strongly suggest which ofthe following diagnoses? A. Miliary"TB involving the kidneys B. Renal cell earcinoma with fang metasiass C. Primary lung cancer with metastasis to the kidaey 1D. Chotioearvinoma with metastasis to lungs and Kidneys E. Acute pyclonephritis with metastatic abscesses inthe lang Answer: B~ note the smoking history, Both lung and renal eanoor ere associated with smoking. Ia this sage, hematuria and the Tuk mass indicates the kidney as the source, Renal metastasi is ‘uncommon, A febrile 23-year-old. woman presents with ex acute onset of right flank pain, suprapubic sliscomfort, dysutia, and inereased frequency of urination. The urinary sediment examination reveals clumps of leukocytes, WBC exsts, occasional RECs, and numerous motile bacteria, The ‘mechanism of this patient's urinary condition is most closely related to. A. arenal stone B. avoending infection C. immunocomplex disease D. drug-induced interstitial nephritis E. _emslogenous spread of infection tothe kidneys Answer: B the patient has acute pyelonephritis (ever, WBC casts, flank pain). This is due to ascending, = infection and vesiooureteral reflux A 25-year-old male presents with a history of hemoptysis and malaise. His blood pressure is 140/90 mm Bg. He has periorbital edema and smoky colored urine, Pertinent laboratory data a1

You might also like